Sunteți pe pagina 1din 98

ID

MERGED UNITY
February 17, 2016

Q588 A 75yo man has left-sided earache and discomfort when he swallows. There is
ulceration at the back of his tongue and he has a palpable non-tender cervical
mass. What is the single most likely dx?
a. Acute mastoiditis
b. Dental abscess
c. Herpes zoster infection
d. Oropharyngeal malignancy
e. Tonsillitis

Clincher(s) Lft sided earache and discomfort on swallowing. Ulcer at the back of tongue
and palpable non tender cervical mass in elderly pt.
A Not associated with swallowing or ulcer of tongue
B
C
D
E No ulcer.
KEY D
Additional
Information
Reference
Dr Khalid/Rabia Pain ear and discomfort during swallowing, ulceration at the back of the
tongue and palpable non tender cervical lymphnode is highly suggestive of
oropharyngeal malignancy. Acute mastoiditis and tonsillitis will not cause
tongue ulcer. Similarly dental abscess will not cause tongue ulcer. In herpes
zoster pain and vesicle will be in the affected nerve distribution].



Q:602 602. A pt presents with progressive visual deterioration. Exam: large, multiple
cotton wool spots in both eyes. What is the single most likely dx?
a. Kaposi’s sarcoma
b. Cryptosporidium
c. CMV infection
d. Pneumocystis carinii infection
e. Cryptococcal infection


Clincher(s) Visual deterioration and cotton wool spots in both eye
A Non tender purplish papilla on face and trunk due to HHV8.
B Mainly cause diarrhoea.
C Mozzarella pizza signs in HIV pts.
D Causes atypical pneumonia
E Causes chronic meningitis with no neck stiffness
KEY C

1
ID MERGED UNITY
February 17, 2016

Additional Cmv retinitis on fundoscopy. Mozarella pizza signs
Information

Reference
Dr Khalid/Rabia Retinitis:
· Retinitis is the most common manifestation of CMV disease in patients who
are HIV positive.
· It presents with decreased visual acuity, floaters, and loss of visual fields on
one side.
· Ophthalmological examination shows yellow-white areas with perivascular
exudates. Haemorrhage is present. Lesions may appear at the periphery of the
fundus, but they progress centrally.
· It begins as a unilateral disease, but in many cases it progresses to bilateral
involvement. It may be accompanied by systemic CMV disease.
Ganciclovir has been used to treat retinitis, but it only slows the progression of
the disease. The optimal treatment is using ganciclovir implants in the
vitreous, accompanied by intravenous ganciclovir therapy.
· Oral ganciclovir may be used for prophylaxis of CMV retinitis. It should not be
used for treatment.



Q:622 A 70yo woman lives in a nursing home following a stroke has developed
reddish
scaly rash on her trunk. She has many scratch marks on her limbs and trunk
with scaling lesions on her hands and feet. What is the single most appropriate
initial tx?
a. Aqueous cream
b. Chlorphenaramine
c. Coal tar
d. 1% hydrocortisone ointment
e. Permethrin

Clincher(s) Elderly lady in nursing home developed the scaly rash
A
B
C

2
ID MERGED UNITY
February 17, 2016

D
E
KEY E
Additional
Information
Reference
Dr Khalid/Rabia


Q:837 A 17yo has acute pain around his right eye, pain on one side of his face and ear
ache too. What is the single most dx?
a. Ear wax
b. Ear foreign body
c. Dental abscess
d. Cellulitis
e. Herpes zoster


Clincher(s) Pain around eye,one side of face and ear.
A Pain will not be on face and eye.
B
C No mention of fever or pain in the gum area or chewing issues.
D
E
KEY E
Additional
Information
Reference
Dr Khalid/Rabia reason : acute onset + unilatral with sensory nerve distribution
THIS PT IS IN the pre eruptive phase >>> no blisters yet



Q:896 What is the mode of spread of chickenpox?
a. Airborne
b. Close contact
c. Fecal-oral
d. Blood
e. Vector


Clincher(s) Mode of spread
A
B
C

3
ID MERGED UNITY
February 17, 2016

D
E In epidemiology, a vector is any agent (person, animal, or microorganism) that
carries and transmits an infectious pathogen into another living organism like
mosquitos
KEY A
Additional Incubation period is 10-21 days. Tx is conservative.
Information It's infective 4 days prior to vesicles eruption until one wk after the last
vesicles scab.
Reference
Dr Khalid/Rabia


Q:905 young girl returns from holidays in Spain. She complains of discharge from her
ear and complains of tragal tenderness. Exam: tympanic membance normal.
Aural toilet has been done. What is the next appropriate med?
a. Antibiotic PO
b. Antibiotic IV
c. Steroid PO
d. Steroid drop
e. Antibiotic drop with steroid


Clincher(s) Otitis externa picture
A
B
C
D
E
KEY E
Additional
Information
Reference
Dr Khalid/Rabia Antibiotic drop with steroid. [Discharge from ear and tragal tenderness are
features of otitis externa. Key treatment is aural toileting. Drop advised is
Sofradex (Framycetin + dexamethasone) 9th edition, page 542].


Q:1358 3yo boy presents with difficulty in walking and skin lesions. What is
the most likely causative agent?
a. Strep pyogenes
b. Rubella virus
c. Parvovirus
d. Papovirus
e. Paramyxovirus

4
ID MERGED UNITY
February 17, 2016


Clincher(s) Skin lesions and difficulty in walking
A
B
C Teenagers and adults may present with a self-limited arthritis. It manifests in
painful swelling of the joints that feels similar to arthritis. Older children and
adults with fifth disease may have difficulty in walking and in bending joints
such as wrists, knees, ankles, fingers, and shoulders
D
E
KEY C parvovirus
Additional Fifth disease (also called erythema infectiosum) is caused by
Information
parvovirus B19. A human virus, parvovirus B19 is not the same,parvovirus
that veterinarians may be concerned about in pets, especially dogs, and it
cannot be passed from humans to animals or vice versa.

Fifth disease starts with a low-grade fever, headache, rash, and cold-like
symptoms, such as a runny or stuffy nose. These symptoms pass, then a few
days later the rash appears. The bright red rash most commonly appears in
the face, particularly the cheeks. This is a defining symptom of the infection
in children (hence the name “slapped cheek disease"). Occasionally the rash
will extend over the bridge of the nose or around the mouth. In addition to
red cheeks, children often develop a red, lacy rash on the rest of the body,
with the upper arms, torso, and legs being the most common locations. The
rash typically lasts a couple of days and may itch; some cases have been
known to last for several weeks. Patients are usually no longer infectious
once the rash has appeared.

Teenagers and adults may present with a self-limited arthritis. It manifests in


painful swelling of the joints that feels similar to arthritis. Older children and
adults with fifth disease may have difficulty in walking and in bending joints
such as wrists, knees, ankles, fingers, and shoulders.


Reference
Dr Khalid/Rabia


Q:1364 30yo pt came to the OPD with complaint of breathlessness and dry
cough. He has lost 5kgs in 2m. He is an IV drug abuser. Inv: CXR=bilateral
interstitial shadowing. What is the single most likely causative organism?
a. Klebsiella
b. TB
c. Chlamydia pneumonia
d. PCP

5
ID MERGED UNITY
February 17, 2016

e. Chlamydia psitacci


Clincher(s) Breathlessness ,IV drug abuser and bilateral interstitial shadowing
A
B
C
D Above mentioned presentation points towards HIV patients and most common
organism causing is PCP
E
KEY D
Additional IV drug abuser is a clincher pointing towards HIV. Most common infections in
Information
HIV pts are caused by PCP.

HIV: Pneumocystis jiroveci pneumonia

Whilst the organism Pneumocystis carinii is now referred to as Pneumocystis

jiroveci, the term Pneumocystis carinii pneumonia (PCP) is still in common


use Pneumocystis jiroveci is an unicellular eukaryote, generally classified as a

fungus but some authorities consider it a protozoa

· PCP is the most common opportunistic infection in AIDS

· all patients with a CD4 count < 200/mm� should receive PCP prophylaxis

Features

· dyspnoea

· dry cough

· fever

· very few chest signs

Pneumothorax is a common complication of PCP.

Extra pulmonary manifestations are rare (1-2% of cases), may cause

· hepatosplenomegaly

· lymphadenopathy

6
ID MERGED UNITY
February 17, 2016

· choroid lesions

Investigation

· CXR: typically shows bilateral interstitial pulmonary infiltrates but can


present with other x-ray findings e.g. lobar consolidation. May be normal

· exercise-induced desaturation

· sputum often fails to show PCP, bronchoalveolar lavage (BAL) often needed

to demonstrate PCP (silver stain shows characteristic cysts)

Management

cotrimoxazole

· IV pentamidine in severe cases

· steroids if hypoxic (if pO2 < 9.3kPa then steroids reduce risk of respiratory

failure by 50% and death by a third)


Reference
Dr Khalid/Rabia


Q:1366 . A man rescued from a building on fire presented with unconsciousness
without any evidence of burns or external injury or soot. What would you do
next?
a. 100% oxyen inhalation
b. 24% oxygen by mask
c. Hyperbaric oxygen in a hyperbaric chamber
d. Intubation
e. Refer to specialist unit

Clincher(s) Unconscious without evidence of burn soot n external injury
A

7
ID MERGED UNITY
February 17, 2016

B
C
D
E
KEY A
Additional I am confused about this one as we should always be following ABC first, but
Information
rule of thumb for securing an airway is GCS less than 8 or a rapid falling GCS.
And there is no soot or burn, obstruction due to laryngeal edema is less
likely. Unconcioussness does not mean a low GCS too low that intubation is
indicated. As it seems to be the case of CO poisoning, First give him isobaric
Oxygen via a facemask and calculate his GCS ( by giving painful stimuli etc )

and then try securing his airway via a LMA or Guedel's airway or an ETT if
expertise is available


Reference
Dr Khalid/Rabia


Q:1377 40yo woman with breast cancer has back pain which keep her awake
at night. She blames it on a gym session she had 2wks ago. She now has
difficulty in climbing stairs. There is tenderness over the right thoracic spine.
She has diminished fine touch and temp sensation in her right foot.
What is the single most appropriate inv?
a. Bone density scan
b. CT head
c. MRI spine
d. Nuclear bone scan
e. XR thoracolumbar spine

Clincher(s) Breast cancer patient ,back ache and diminished touch and temp sensation in
right foot
A
B
C
D
E
KEY MRI spine
Additional This was a case of spine metastasis (vertebra) and stress during exercising in
Information gym caused collapsing fracture of vertebra which lead to spinal cord
compression. X-ray will only show vertebral fracture and not the
compression. As there is neuro features so to deliniate the spinal cord
compression we have to do MRI.

8
ID MERGED UNITY
February 17, 2016


Reference
Dr Khalid/Rabia


Q:1380
A pt admitted due to repeated attacks of pancreatitis presents with
dementia and loss of proprioception in the legs. What is the most appropriate
tx?
a. Thiamine
b. Pyridoxine
c. Cobolamin
d. Lipase
e. Antibiotics

Clincher(s) Pancreatitis with dementia and loss of proprioception
A Is the answer as the presentation goes in favour of a person being alcoholic
and Dec B 12 so thiamine
B
C
D
E
KEY C pancreatic enzyme activates IF and lack of it will lead to vit B12 def
Additional Pancreatic enzymes in the duodenum cleave off the R-binders(which come
Information from salivary glads)

from Vit-B12, which then combines with IF and this complex is absorbed in
terminal ileum.

Pancreactic enzyme deficiency therefore leads to impaired absorption of Vit


B12 and sub-acute combined degeneration of spinal cord Alcoholic +
pancreatitis more in favor of B12 Alcoholic only think of thiamine


Reference
Dr Khalid/Rabia


Q:1382
During a laparoscopic procedure, a trochar is inserted halfway between
the umbilicus and the ant superior iliac spine. What are the structures most
likely to be pierced?
a. Rectus sheath
b. Linea alba
c. External oblique aponeurosis
d. Internal oblique and transverse abdominal

9
ID MERGED UNITY
February 17, 2016

e. Both C and D

Clincher(s)
A
B
C
D
E
KEY D but normally all those two plus external oblique muscle will be pierced.
Additional There is a confusion regarding the answer of this question. But, I think the
Information
answer should be E considering the insertions of ext oblique.


Reference
Dr Khalid/Rabia

Q:1387
A 55yo man presented with hot, raised, tender area of skin on his right
leg. He is febrile with rigors. He has been started on flucloxacillin. What other
meds will you add?
a. Ciprofloxacin
b. Gentamicin
c. Metronidazole
d. Benzylpenicillin
e. Ceftriaxone

Clincher(s) Signs of inflammation on flucoxacillin
A
B
C
D
E
KEY D
Additional Cellulitis is a term used to describe an inflammation of the skin and
Information subcutaneous tissues, typically due to infection by Streptococcus pyogenes
or Staphylcoccus aureus.

Features · commonly occurs on the shins erythema, pain, swelling

· there may be some associated systemic upset such as fever

Management

The BNF recommends flucloxacillin as first-line treatment for mild/moderate


cellulitis. Clarithromycin or clindamycin is recommend in patients allergic to

10
ID MERGED UNITY
February 17, 2016

penicillin. Many local protocols now suggest the use of oral clindamycin in
patients who have failed to respond to flucloxacillin. Severe cellulitis should
be treated with intravenous benzylpenicillin +

flucloxacillin.


Reference
Dr Khalid/Rabia


Q:1388 65yo man has incurable bronchial cancer. He is unable to cough up
his secretions. This is leading to a distressing cough. Which of the following
drugs is most likely to help him?
a. Scopolamine
b. Xanomeline
c. Aceclidine
d. Pilocarpine
e. Cevimiline

Clincher(s) Bronchial cancer and cough
A Hyoscine
B
C
D
E
KEY A
Additional Scopolamine : Antisecretory
Information
Xanomeline : Trial in Alzheimer’s and Pscizophrenia

Aceclidine : tx of narrow angle glaucoma

Cevimiline : Dry mouth


Reference
Dr Khalid/Rabia


Q:1389 pt presented after eating a seafood dish at a local restaurant. He
complains of difficulty in breathing. His speech is slurred and his
BP=85/55mmHg. What would be the most appropriate next step?
a. IV adrenaline
b. IM adrenaline
c. SC adrenaline

11
ID MERGED UNITY
February 17, 2016

d. PO chlorpheniramine
e. IV chlorpheniramine

Clincher(s) Consumption of sea food and difficulty in breathing and slurred speech and
decrease BP
A IV and IM given in emergencies but iM is more preferred because IV causes
increase chances of fatal arrhythmias so if required needs diluted infusion and
constant monitoring of heart function
B Is preferred as it has least fatal complications and reaches peak level soon
C Sc not given as it causes cutaneous Vaso constriction which will cause delay
absorption
D
E
KEY B
Additional Anaphylaxis may be defined as a severe, life-threatening, generalised or
Information systemic hypersensitivity reaction. Anaphylaxis is one of the few times when
you would not have time to look up the dose of a medication. The
Resuscitation Council guidelines on anaphylaxis have recently been updated.
Adrenaline is by far the most important drug in anaphylaxis and should be
given as soon as possible. The recommended doses for adrenaline,
hydrocortisone and chlorphenamine are as follows:

Table for adrenaline ,hydrocortisone,chlorpheneramine

Adrenaline can be repeated every 5 minutes if necessary. The best site for IM

12
ID MERGED UNITY
February 17, 2016

injection is the anterolateral aspect of the middle third of the thigh.

Common identified causes of anaphylaxis

· food (e.g. Nuts) - the most common cause in children

· drugs

· venom (e.g. Wasp sting

Adrenaline can be repeated every 5 minutes if necessary. The best site for IM

injection is the anterolateral aspect of the middle third of the thigh.

Common identified causes of anaphylaxis

· food (e.g. Nuts) - the most common cause in children

· drugs

· venom (e.g. Wasp sting


Reference
Dr Khalid/Rabia Rabia

Q:1400 A young anxious mother of a 10m boy comes to you and requests a
test for CF. What is the most
appropriate inv?
a. Sweat test
b. Heel prick test
c. Breath test
d. CXR


Clincher(s) Test for CF ,cystic fibrosis
A
B
C Alcohol detection
D

13
ID MERGED UNITY
February 17, 2016

E
KEY A
Additional CF is an autosomal recessive disease caused by mutations in the CF
Information transmembrane conductance regulator (CFTR) gene, on chromosome 7. The
only risk factor is a family history of the condition.

Signs

These may include:

·Finger clubbing.

· Cough with purulent sputum.

· Crackles.

Wheezes (mainly in the upper lobes).

·Forced expiratory volume in one second (FEV1) showing obstruction.

Babies diagnosed with CF will usually have no signs or symptoms.

Investigations

· Sweat testing confirms the diagnosis and is 98% sensitive.

Chloride concentration >60 mmol/L with sodium concentration lower than

that of chloride on two separate occasions.

·Molecular genetic testing for CFTR gene.

· Sinus X-ray or CT scan - opacification of the sinuses is present in

almost all patients with CF.

·CXR or CT of thorax.

·Lung function testing - spirometry is unreliable before 6 years.

· Sputum microbiology - common pathogens include Haemophilus

influenzae,Staphylococcus aureus, Pseudomonas aeruginosa,

Burkholderia cepacia, Escherichia coli and Klebsiella pneumoniae.

14
ID MERGED UNITY
February 17, 2016

·Various blood tests including FBC, U&Es, fasting glucose, LFTs and

vitamin A, D and E levels are usually performed.

Management of cystic fibrosis involves a multidisciplinary approach

Key points

· regular (at least twice daily) chest physiotherapy and postural drainage.

Parents are usually taught to do this. Deep breathing exercises are also

useful

· high calorie diet, including high fat intake*

· vitamin supplementation

· pancreatic enzyme supplements taken with meals

· heart and lung transplant

exclusion of other options:

Heel prick test= this test is usually done on the 5th to 6th day of life. A blood
spot is taken

from baby’s heel it is done for screening diseases like sickle cell disease,
cystic fibrosis,

congenital hypothyroidism, phenylketonuria, homocystinuria etc.In cystic


fibrosis the heel

prick test detects a chemical called immunoreactive trypsinogen.

CXR=not diagnostic in CF

Cystic Fibrosis:

CF is an autosomal recessive disease caused by mutations in the CF


transmembrane econductance regulator (CFTR) gene, on chromosome 7.
CFTR is an ATP-responsive chloride channel that also affects other cellular
activities, such as sodium transport across the respiratory epithelium,
composition of cell surface glycoprotein and antibacterial defences.

15
ID MERGED UNITY
February 17, 2016

Clinical features:

neonates= failure to thrive,meconium ileus,rectal prolapse

children and adults= respiratory: recurrent chest infections ( recurrent lower


respiratory tract

infection (LRTI) with chronic sputum production is the most common


presentation)

GIT: pancreatic insufficiency, gallstones, cirrhosis etc

others= male infertility,vasculitis,nasal


polyps,arthritis,osteoporosis,hypertrophic pulmonary osteoarthropathy
Signs= finger clubbing,cyanosis,bilateral coarse crackles

Investigations=

DIAGNOSIS by SWEAT TEST= . Chloride concentration >60 mmol/L with


sodium concentration lower than that of chloride on two separate occasions.
Molecular genetic testing for CFTR gene

Sputum microbiology - common pathogens include Haemophilus


influenzae,Staphylococcus aureus, Pseudomonas aeruginosa, Burkholderia
cepacia, Escherichia coli and Klebsiella pneumoniae.

Treatment=

Patient care is most effective when provided in specialist centres by


multidisciplinary teams.

symptomatic treatment for instance, respiratory= chest physiotherapy

GIT= pancreatic enzyme replacement, fat sol vitamin supplementation etc


Reference
Dr Khalid/Rabia Rabia


Q:1426 40yo pt came to OPD with complaint of fever, pleuritic chest pain, productive
cough and painful vesicles around the lips. Exam: temp=38C. He has a hx of
splenectomy last yr. What is the single most likely causative organism?
a. Pneumococcal pneumonia
b. Staphylococcus

16
ID MERGED UNITY
February 17, 2016

c. Klebsiella
d. Streptococcus
e. Chlamydia psitacci


Clincher(s) Splenectomy
A Most common in splenectomised patients
B
C
D
E
KEY A
Additional Answer= A Pneumococcal pneumonia. the patient has a history of
Information splenectomy and pts with splenectomy are more prone to develop
pneumococcus pneumonia that is why they are given vaccines
prophylactically.

Community acquired pneumonia (CAP) may be caused by the following


infectious agents: · Streptococcus pneumoniae (accounts for around 80% of
cases)

Haemophilus influenzae

· Staphylococcus aureus: commonly after the 'flu

· atypical pneumonias (e.g. Due to Mycoplasma pneumoniae)

· viruses

Klebsiella pneumoniae is classically in alcoholics ,Streptococcus pneumoniae


(pneumococcus) is the most common cause of community-acquired
pneumonia

Clinical features

Symptoms: Fever, rigors, malaise, anorexia, dyspnoea, cough, purulent


sputum, Haemoptysis, and pleuritic pain.

Signs: Pyrexia, cyanosis, confusion (elderly usually), tachypnoea, tachycardia,


hypotension, signs of consolidation (diminished expansion, dull percussion
note, tactile vocal Fremitus/vocal resonance, bronchial breathing), and a
pleural rub.

CXR: lobar or multilobar infiltrates, cavitation or pleural effusion.

Assess oxygenation: oxygen saturation (ABGs if SaO2 <92% or severe

17
ID MERGED UNITY
February 17, 2016

pneumonia) and

Blood tests: FBC, U&E, LFT, CRP, blood cultures. Sputum for microscopy and
culture.

Pleural fluid may be aspirated for culture.

Consider bronchoscopy and bronchoalveolar lavage if patient is


immunocompromised or on

Management:

CURB-65 criteria of severe pneumonia

· Confusion (abbreviated mental test score <= 8/10)

· Urea > 7 mmol/L

· Respiratory rate >= 30 / min

· BP: systolic <= 90 or diastolic <= 60 mmHg

· age >= 65 years

· Patients with 3 or more (out of 5) of the above criteria are regarded as


having a severe pneumonia

The British Thoracic Society published guidelines in 2009:

· low or moderate severity CAP: oral amoxicillin. A macrolide should be


added for patients admitted to hospital

· high severity CAP: intravenous co-amoxiclav + clarithromycin OR


cefuroxime +clarithromycin OR cefotaxime + clarithromycin

· the current BNF has slightly different recommendations for high severity
CAP: intravenous benzylpenicillin + clarithromycin OR benzylpenicillin +
doxycycline. For 'life-threatening' infections the BNF recommends the same
as the BTS guidelines for high-severity CAP
Reference
Dr Khalid/Rabia


Q:

Clincher(s)
A

18
ID MERGED UNITY
February 17, 2016

B
C
D
E
KEY
Additional
Information
Reference
Dr Khalid/Rabia


Q:

Clincher(s)
A
B
C
D
E
KEY
Additional
Information
Reference
Dr Khalid/Rabia


Q:

Clincher(s)
A
B
C
D
E
KEY
Additional
Information
Reference
Dr Khalid/Rabia


Q:

Clincher(s)
A

19
ID MERGED UNITY
February 17, 2016

B
C
D
E
KEY
Additional
Information
Reference
Dr Khalid/Rabia

Q:

Clincher(s)
A
B
C
D
E
KEY
Additional
Information
Reference
Dr Khalid/Rabia


Q:

Clincher(s)
A
B
C
D
E
KEY
Additional
Information
Reference
Dr Khalid/Rabia


Q:

Clincher(s)
A
B

20
ID MERGED UNITY
February 17, 2016

C
D
E
KEY
Additional
Information
Reference
Dr Khalid/Rabia

Q:1019 1019. A 68yo woman presents to the ED with confusion. Temp=39.3C and
productive cough. Sputum is rusty colored after 2 days. CXR shows right lower
lobe consolidation. What is the most likely organism?
a. Streptococcus pneumonia
b. Staphylococcus aureus
c. Coxiella burnetti
d. Mycoplasma pneumonia

Clincher(s)
A
B
C
D
E
KEY A
Additional Symptoms
Information • Fever, rigors, malaise, anorexia, dyspnoea, cough, purulent sputum
(classically
‘rusty’ with pneumococcus), haemoptysis, and pleuritic chest pain.ohcm 826
Reference
Dr Khalid/Rabia Ans: A
Reason:most common pathogen causing pneumonia in elderly, Rusty Coloured
Sputum, lobar cosolidation on cxr.
treatment : Amoxixillin 5-8 days
benzylpenicillin, cephalosporin


Q:1048 1048. A lady developed breast abscess after delivery. What is the most likely
organism?
a. Staph aureus
b. Staph albus
c. GBS
d. Strep pyogenes
e. Strep faecalis

Clincher(s)

21
ID MERGED UNITY
February 17, 2016

A
B
C
D
E
KEY A
Additional
Information
Reference
Dr Khalid/Rabia Dx: puerperal mastitis leading to abscess
The usual infecting organism is Staphylococcus aureus, although it may also be
Staphylococcus albus and streptococci. Meticillin-resistant Staphylococcus
aureus (MRSA) infection is increasing, and may be more common in women
who have had a caesarean section.
Dx: clinical
Rx
· Incision and drainage of abscess with cavity packed open with gauze is
recommended if the overlying skin is thin or necrotic.
· Parenteral antibiotics should be administered at the same time, with added
coverage for anaerobic bacteria. Fluid from the abscess should be cultured,
and results used to determine ongoing antibiotic treatment.
· Needle aspiration of the abscess, repeated every other day until the pus no
longer accumulates, has been suggested as an alternative to open drainage.
· In some cases breast-feeding may have to cease until the abscess is
successfully treated, but can usually resume later


Q:1186 1186. A homeless lady presents with cough and fever. She complains of night
sweats and weight loss.
CXR has been done and shows opacity. What is the next appropriate
management?
a. AFB
b. Mantoux test
c. IFN gamma testing
d. Bronchoscopy

Clincher(s)
A
B
C
D
E
KEY A
Additional
Information

22
ID MERGED UNITY
February 17, 2016

Reference
Dr Khalid/Rabia a. AFB
Mantoux test can only tell you "the exposure which may be the past", can't
confirm active disease
INVESTIGATIONS:
1- CXR
2- Sputum for AFB/culture
3- BAL




Q:1190 1190. A 36yo woman contacts the police to notify them she was responsible
for a recent disastrous flood with loss of lives. What kind of delusions is she
suffering from?
a. Persecutory
b. Poverty
c. Guilt
d. Nihilistic
e. Reference

Clincher(s)
A
B
C
D
E
KEY C
Additional
Information
Reference
Dr Khalid/Rabia


Q: 1191. A 27yo man presents with symptoms characterized by alternating mood
swings a/w flight of
ideas, elation, over activity and disinhibition, or low mood with lack of energy
and social
withdrawal. What is the most probable dx?
a. Bipolar affective disorder
b. Dysthymia
c. Mania
d. Hypomania
e. Cyclothymia

23
ID MERGED UNITY
February 17, 2016

Clincher(s)
A
B
C
D
E
KEY A
Additional
Information
Reference
Dr Khalid/Rabia 1191. A 27yo man presents with symptoms characterized by alternating mood
swings a/w flight of ideas, elation, over activity and disinhibition, or low mood
with lack of energy and social withdrawal. What is the most probable dx?
a. Bipolar affective disorder
b. Dysthymia
c. Mania
d. Hypomania
e. Cyclothymia

a. Bipolar affective disorder
Basic definition

Q:1195 1195. A terminally ill pt with metastatic carcinoma presents with dysphagia
and difficulty in
swallowing. What is the best possible tx?
a. Nystatin suspension
b. Amphotericin B IV
c. PO fluconazole
d. Cotrimazole
e. Analgesic

Clincher(s)
A
B
C
D
E
KEY C
Additional
Information
Reference
Dr Khalid/Rabia Oesophageal candidiasis is the most common cause of oesophagitis in patients
with HIV. It is generally seen in patients with a CD4 count of less than 100.
Typical symptoms include dysphagia and odynophagia. Fluconazole and
itraconazole are first-line treatments

24
ID MERGED UNITY
February 17, 2016


Q: 1204. A young boy presented with peri-oral blisters. Some of which are
weeping and others are
crusted. What is the single most appropriate dx?
a. Impetigo
b. Varicella zoster
c. Shingles
d. Scabies
e. Herpes simplex

Clincher(s)
A
B
C
D
E
KEY
Additional
Information
Reference A
Dr Khalid/Rabia a. Impetigo

Non-bullous - tiny pustules change into honey coloured crusted plaques <2cm

Mostly on exposed area like face extremities especially where bites, abrasions,
lacerations, scratches, burns or trauma have occurred.

Spreads rapidly

Some itching may be present

Bullous - thin roof, so rupture spontaneously
Occur on face, trunk, extremities, buttocks or perineal region



Q:1205 1205. A 39yo man comes with umbilicated papules on his face. His CD4 count
is measured to be 35.
What is the single most appropriate option?
a. Mycobacterium avium
intercellular
b. CMV
c. Streptokinase
d. Toxoplasmosis
e. Pneumocystis jerovici
f. Moluscum contagiosum

25
ID MERGED UNITY
February 17, 2016


Clincher(s)
A
B
C
D
E
KEY F
Additional
Information
Reference
Dr Khalid/Rabia f. Moluscum contagiosum

Viral skin infection (Opportunistic) in immunocompromised individuals

Firm, smooth, Umbilicated papules

Skin coloured, white, translucent or slightly yellow

Occurs in clusters, in children, found on trunk or extremities

In adults, found on lower abdomen, inner thighs or genital region




Q:1207 1207. An 18yo man has a smooth, tender swelling extending from the ear to
the angle of the jaw of sudden onset. Temp=38.5C. What is the single most
likely dx?
a. Dental caries
b. Mumps
c. OE
d. OM

Clincher(s)
A
B
C
D
E
KEY
Additional
Information
Reference
Dr Khalid/Rabia b. Mumps

26
ID MERGED UNITY
February 17, 2016


Fever, myalgia , malaise painful parotid swelling which becomes bilateral in
70%

Spread - droplets/saliva

Incubation period - 14-21 days

Complications - rare but may lead to orchitis arthritis, meningitis



Q: 1218. A 10yo boy presents with irritability, sudeen onset of pain and discharge
from the right ear.
Which antibiotic would be the 1st line of tx?
a. Amoxicillin
b. Ciprofloxacillin
c. Flucloxacillin
d. Ceftazidime
e. Benzyl penicillin

Clincher(s)
A
B
C
D
E
KEY
Additional
Information
Reference
Dr Khalid/Rabia . Amoxicillin

Acute Otitis Media is inflammation of middle ear presenting with pain,
malaise, irritability, fever, vomiting. Red and bulging ear drums may be
present. Also, hearing loss may be present.

Usually resolves spontaneously without specific treatment. Analgesia may be
required

In cases with symptoms longer than 2-3 days, children <2 years with bilateral
disease or bulging drum, any child with Otorrhoea, antibiotics are required. In
such cases, a five day course of Amoxicillin is prescribed. In case the patient is
allergic to penicillin, erythromycin

27
ID MERGED UNITY
February 17, 2016

1237, 1245, 1248, 1291


Q:1237 A 6wk baby has been dx as HIV+ve. Which immunization plan will you opt for
him?
a. Don’t give any vaccine
b. Give all vaccines except live attenuated vaccines
c. Give only BCG vaccine
d. Give all vaccines except BCG vaccine

Clincher(s)
A Don’t give any vaccine
B Give all vaccine except live attenuated vaccines
C Give only BCG vaccine
D Give all vaccine except BCG vaccine
E
KEY B
Additional Vaccines that can be used in Vaccines that can be Contraindicated in
Information all HIV-infected adults used if CD4 > 200 HIV-infected adults
Hepatitis A
Hepatitis B
Haemophilus influenzae B
(Hib)
Influenza-parenteral Measles, Mumps, Cholera CVD103-HgR
Japanese encephalitis Rubella (MMR) Influenza-intranasal
Meningococcus-MenC Varicella Poliomyelitis-oral (OPV)
Meningococcus-ACWY I Yellow Fever Tuberculosis (BCG)
Pneumococcus-PPV23
Poliomyelitis-parenteral (IPV)
Rabies
Tetanus-Diphtheria (Td)

Reference
Dr Khalid/Rabia Vaccination in Children with Immunodeficiency states (D)
- An asymptomatic child with HIV infection can be given any vaccine except
BCG.
- BCG vaccine is contraindicated in symptomatic HIV-infected child as well
along with a few live attenuated vaccines.
- In this scenario the child has just been diagnosed with an HIV infection which
most probably means that he is asymptomatic. So the answer would be D, Give
all vaccines except BCG.


Q:1245 A 16yo boy came home from boarding school with a cough. His CXR showed
bilateral consolidations. What is the most likely organism which would have
caused his symptoms?
a. Legionella pneumophilia

28
ID MERGED UNITY
February 17, 2016

b. Mycoplasma pneumonia
c. Mycobacterium TB
d. Pneumocystis jiroveci
e. Pseudomonas aeruginosa

Clincher(s) A
A Legionella pneumophilia-
B Mycoplasma pneumonia
C Mycobacterium TB
D Pneumocystis jiroveci
E Pseudomonas aeruginosa- Colonises Cystic Fibrosis
KEY A
Additional Pneumonia in children
Information

The British Thoracic Society published guidelines in 2002 on the management
of community acquired pneumonia in childhood. Key points are summarised
below:

Treatment

• in children < 5 years old amoxicillin is first line


• in children > 5 years old a macrolide such as erythromycin or
clarithromycin is first line as there is a greater prevalence of
mycoplasma pneumonia in this age group

Pneumonia: prognostic factors


The British Thoracic Society recommends that patients should be assessed for
the severity of their pneumonia using several core prognostic features (CURB-
65 score).

The CURB-65 score is as follows:


Criterio Marker
n
C Confusion (abbreviated mental test score <= 8/10)
U Urea >7 mmol/L
R Respiration rate >= 30/min
Blood pressure: systolic <= 90 mmHg and/or diastolic <= 60
B
mmHg
65 Aged >= 65 years

Patients with a CURB-65 score of 0 should be managed in the community.

29
ID MERGED UNITY
February 17, 2016


Patients with a CURB-65 score of 1 should have their Sa02 assessed which
should be >92% to be safely managed in the community and a CXR performed.
If the CXR shows bilateral/multilobar shadowing hospital admission is advised.

Patients with a CURB-65 score of 2 or more should be managed in hospital as
this represents a severe community acquired pneumonia.

The CURB-65 score also correlates with an increased risk of mortality at 30
days with patients with a CURB-65 score of 4 approaching a 30% mortality rate
at 30 days.

Other factors associated with a poor prognosis include:

• presence of coexisting disease


• hypoxaemia (pO2 < 8 kPa) independent of FiO2

Mycoplasma pneumoniae


Mycoplasma pneumoniae is a cause of atypical pneumonia which often affects
younger patients. It is associated with a number of characteristic complications
such as erythema multiforme and cold autoimmune haemolytic anaemia.
Epidemics of Mycoplasma pneumoniae classically occur every 4 years. It is
important to recognise atypical pneumonias as they may not respond to
penicillins or cephalosporins due to it lacking a peptidoglycan cell wall.

Features

• the disease typically has a prolonged and gradual onset


• flu-like symptoms classically precede a dry cough
• bilateral consolidation on x-ray
• complications may occur as below


Complications

• cold agglutins (IgM) may cause an haemolytic anaemia,


thrombocytopenia
• erythema multiforme, erythema nodosum
• meningoencephalitis, Guillain-Barre syndrome
• bullous myringitis: painful vesicles on the tympanic membrane
• pericarditis/myocarditis
• gastrointestinal: hepatitis, pancreatitis
• renal: acute glomerulonephritis

30
ID MERGED UNITY
February 17, 2016

Investigations

• diagnosis is generally by Mycoplasma serology


• positive cold agglutination test


Management

• erythromycin/clarithromycin
• tetracyclines such as doxycycline are an alternative

Comparison of Legionella and Mycoplasma pneumonia

HIV: Pneumocystis jiroveci pneumonia


Whilst the organism Pneumocystis carinii is now referred to as Pneumocystis
jiroveci, the term Pneumocystis carinii pneumonia (PCP) is still in common use

• Pneumocystis jiroveci is an unicellular eukaryote, generally classified as


a fungus but some authorities consider it a protozoa
• PCP is the most common opportunistic infection in AIDS
• all patients with a CD4 count < 200/mm� should receive PCP
prophylaxis


Features

• dyspnoea
• dry cough
• fever
• very few chest signs

31
ID MERGED UNITY
February 17, 2016


Pneumothorax is a common complication of PCP.

Extrapulmonary manifestations are rare (1-2% of cases), may cause

• hepatosplenomegaly
• lymphadenopathy
• choroid lesions


Investigation

• CXR: typically shows bilateral interstitial pulmonary infiltrates but can


present with other x-ray findings e.g. lobar consolidation. May be
normal
• exercise-induced desaturation
• sputum often fails to show PCP, bronchoalveolar lavage (BAL) often
needed to demonstrate PCP (silver stain shows characteristic cysts)


Management

• co-trimoxazole
• IV pentamidine in severe cases
• steroids if hypoxic (if pO2 < 9.3kPa then steroids reduce risk of
respiratory failure by 50% and death by a third)

32
ID MERGED UNITY
February 17, 2016

© Image used on license from Radiopaedia

CT scan showing a large pneumothorax developing in a patient with


Pneumocystis jiroveci pneumonia


Reference
Dr Khalid/Rabia - History of living in boarding
- Bilateral consolidations
- Dry Cough
- Crepitations on auscultation
- Fever, anorexia and lethargy
The above points point clearly towards Legionella (A)
- Mycoplasma does not cause bilateral consolidation
- Staphylococcus causes bilateral basal cavitation and presents with a
productive cough.
- Streptococcus shows lobar consolidation and presents with a productive
cough.



Q:1248 A young man returns to his hostel and gets headache and lethargy. Now
presents with fever. There are crepitations on the auscultation of lung. What is
the most likely organism which would have caused his symptoms?
a. Legionella pneumonia

33
ID MERGED UNITY
February 17, 2016

b. Mycoplasma
c. Staphylococcus
d. Streptococcus

Clincher(s)
A
B
C
D
E
KEY A
Additional
Information
Reference
Dr Khalid/Rabia - History of living in boarding
- Bilateral consolidations
- Dry Cough
- Crepitations on auscultation
- Fever, anorexia and lethargy
The above points point clearly towards Legionella (A)
- Mycoplasma does not cause bilateral consolidation
- Staphylococcus causes bilateral basal cavitation and presents with a
productive cough.
- Streptococcus shows lobar consolidation and presents with a productive
cough


Q:1291 At birth, a baby boy at 38wks GA weighs 1.8kgs. He has hepato-splenomegaly
and a rash. Blood test show raised level of bilirubin and liver enzymes. What is
the most likely dx?
a. Galactosemia
b. Biliary atresia
c. G6PD deficiency
d. Rh incompatibility
e. Congenital viral infection

Clincher(s)
A
B
C
D
E
KEY E
Additional
Information

34
ID MERGED UNITY
February 17, 2016

Reference
Dr Khalid/Rabia Congenital rubella....blueberry muffin rash with extra medullary hematopoiesis
leading to hepatosplenomegaly, deranged liver enzymes with low birth weight

Congenital infections

The major congenital infections encountered in examinations
are rubella, toxoplasmosis and cytomegalovirus

Cytomegalovirus is the most common congenital infection in the UK. Maternal
infection is usually asymptomatic

Rubella Toxoplasmosis Cytomegalovirus

Characteri Sensorineural Cerebral Growth


stic deafness calcification retardation
features Congenital cataracts Chorioretinitis Purpuric skin
Congenital heart Hydrocephalus lesions
disease (e.g. patent
ductus arteriosus)
Glaucoma

Other Growth retardation Anaemia Sensorineural


features Hepatosplenomegaly Hepatosplenomeg deafness
Purpuric skin lesions aly Encephalitis/seizu
'Salt and pepper' Cerebral palsy res
chorioretinitis Pneumonitis
Microphthalmia Hepatosplenomeg
Cerebral palsy aly
Anaemia
Jaundice
Cerebral palsy




Q:

Clincher(s)
A
B
C
D
E
KEY

35
ID MERGED UNITY
February 17, 2016

Additional
Information
Reference
Dr Khalid/Rabia

Q:

Clincher(s)
A
B
C
D
E
KEY
Additional
Information
Reference
Dr Khalid/Rabia

Q:

Clincher(s)
A
B
C
D
E
KEY
Additional
Information
Reference
Dr Khalid/Rabia


Q:

Clincher(s)
A
B
C
D
E
KEY
Additional
Information

36
ID MERGED UNITY
February 17, 2016

Reference
Dr Khalid/Rabia


Q:

Clincher(s)
A
B
C
D
E
KEY
Additional
Information
Reference
Dr Khalid/Rabia


Q:

Clincher(s)
A
B
C
D
E
KEY
Additional
Information
Reference
Dr Khalid/Rabia




Q: 821 . A 29yo man presents with hx of cough, weight loss and night sweats. Exam:
pansystolic murmur. What is the most appropriate dx of underlying cause?
a. Malaria
b. HSP
c. HIV
d. Dengue fever


Clincher(s) 29 year old with cough, weight loss, night sweats and pansystolic murmur.

37
ID MERGED UNITY
February 17, 2016

A
B Vasculitis, variant of IgA nephropathy.
C
D
E
KEY C . HIV
Additional Tricuspid regurgitation due to infective endocarditis in IV drug abusers is very
Information common. IV drug abuse can predispose to HIV.
Reference
Dr Khalid/Rabia Patients with HIV are likely to have heart valve disease (tricuspid mc in drug
abusers) leading to pansystolic murmur. All other options do not have direct
relation to valvular disease.



Q: 825 A 67yo man being managed for a malignancy develops neutropenic fever. He
has been commenced on Ticacillin, Tazobactam and Gentamicin. He has also
recently commenced on Meropenem but on the 3rd day his temp still remains
>39C. 2 blood tests and urine cultures show no organism. Inv: Hgb=104g/dl,
WBC=<0.5, Plt=15. What will you do next?
a. Continue IV antibiotics and add oral antifungals
b. Continue antibiotics and add IV antifungals
c. Stop antibiotics
d. Continue only present antibiotics



Clincher(s) 67 years old with malignancy and neutropenia develops fever.
A
B
C
D
E
KEY b. continue antibiotics and add IV fungals
Additional
Information
Reference
Dr Khalid/Rabia


Q: 830 A 32yo man has been to Thailand and returned with cervical lymphadenopathy
and fever. What is he most likely suffering from?
a. HIV
b. EBV
c. Typhoid

38
ID MERGED UNITY
February 17, 2016

d. Measles


Clincher(s)
A
B
C
D
E
KEY key : b
reason : clinical pic fits EBV ( fever + lymphadenopathy )
synonms : glandular fever / infectious mononucleosis

Additional
Information
Reference
Dr Khalid/Rabia Presentation :
• Low-grade fever, fatigue and prolonged malaise.
• Lymphadenopathy, especially neck glands
• Sore throat; tonsillar enlargement is common, classically exudative and
may be massive; palatal petechiae and uvular oedema.
• Fine macular non-pruritic rash
Later signs include:
• Mild hepatomegaly and splenomegaly (splenic enlargement returns
to normal or near normal usually within three weeks after the clinical
presentation) with tenderness over the spleen.
• Jaundice occurs in fewer than 10% of young adults, but in as many as
30% of infected elderly patients.
Investigations :
Heterophile antibodies >>> Following IM caused by EBV, 70-90% of patients
produce IM heterophile antibodies (antibodies against an antigen produced in
one species that react against antigens from other species). The heterophile
antibodies are not specific for the virus. These antibodies can be detected by :
• Paul-Bunnell test
• Monospot® test
EBV-specific antibodies
Management :
Patients are traditionally advised to avoid contact sports for three weeks
because of the risk of splenic rupture
Avoid alcohol for the duration of the illnes
Advise paracetamol for analgesia and control of fever.
No specific antiviral therapy is available.
Short courses of corticosteroids are beneficial for haemolytic anaemia, central
nervous system involvement or extreme tonsillar enlargement.

39
ID MERGED UNITY
February 17, 2016


Q: 912 A 16wk pregnant pt who was exposed to a child with chickenpox came to GP
for help. She was tested –ve for varicella antibody. What is the next most imp
step in management?
a. Reassurance
b. Ig
c. Ig + vaccine
d. Vaccine only
e. Acyclovir


Clincher(s) 16 wk pregnant, exposed to child with chicken pox, tested -ve for varicella
antibody

A
B
C
D
E
KEY B
Additional If the pregnant woman has chickenpox contact abnd is susceptible(confirmed
Information by urgent blood test), varicella zoster immune globulin (VZIG) can be given
within 10 days of exposure.
Reference Ohcs 34
Dr Khalid/Rabia Ig + vaccine or only vaccine are contraindicated during pregnancy
Acyclovir ... there are no signs of infection

Ig is given because there is only history of exposure and there are no signs of
infection yet.




Q: 914 A 5yo boy was brought to GP with high temp and many vesicles on his back.
What is the most appropriate management?
a. Topic acyclovir
b. Oral acyclovir
c. Oral antibiotics
d. Topical steroids
e. None


Clincher(s) 5 year old with high temp and vasicles on back
A
B

40
ID MERGED UNITY
February 17, 2016

C
D
E
KEY e
Additional Chicken pox or vzv .
Information There is no specific treatment for chickenpox, but there are pharmacy
remedies that can alleviate symptoms. These include paracetamol to relieve
fever, and calamine lotion and cooling gels to ease itching.
In most children, the blisters crust up and fall off naturally within one to two
weeks.

Reference Nhs
Dr Khalid/Rabia no treatment is required in children from 28days old to 12years old in chicken
pox.



Q:

Clincher(s)
A
B
C
D
E
KEY
Additional
Information
Reference
Dr Khalid/Rabia

Q:

Clincher(s)
A
B
C
D
E
KEY
Additional
Information
Reference
Dr Khalid/Rabia

41
ID MERGED UNITY
February 17, 2016



Q:

Clincher(s)
A
B
C
D
E
KEY
Additional
Information
Reference
Dr Khalid/Rabia


Q:

Clincher(s)
A
B
C
D
E
KEY
Additional
Information
Reference
Dr Khalid/Rabia

Q: 628 A lady who works at a nursing home presents with itching. Exam: linear tracks
on the wrist. She
says that 2d ago she had come in contact with a nursing home inmate with
similar symptoms.
What is the mechanism of itching?
a. Infection
b. Destruction of keratinocytes
c. Allergic reaction
d. Immunosuppression
e. None


Clincher(s) Nursing home worker, itching linear tracksn on wrists, contact with inmate
with similar symptoms

42
ID MERGED UNITY
February 17, 2016

A
B
C
D
E
KEY C
Additional Scabies. pruritis due to allergic reaction.
Information
Reference
Dr Khalid/Rabia


Q: 631 A 28yo man presents with a maculopapular rash over his trunk and palms. He
also has
numerous mouth ulcers. He had a penile ulcer which healed 2wks ago. What
will you do to
confirm the dx?
a. PCR for treponemal and non-treponemal antibiodies
b. Dark ground microscopy from mouth ulcer
c. Blood culture for treponema
d. Dengue fever



Clincher(s) Maculopapular rash on trunk and palms. Mouth ulcers. Healed penile ulcer.
A
B Dark ground microscopy is done of chancre fluid
C Both antibody tests are positive in serology
D
E
KEY a. PCR for treponemal and nontreponemal antibodies

Additional Secondary syphilis
Information
Reference
Dr Khalid/Rabia because it has now progressed to secondary syphilis and the investigation of
choice is PCR
Dark ground microscopy is done from the chancre fluid and not from mouth
ulcers
stage 1/ primary= dark field microscopy. 2= treponeme specific and non-
antibodies, treponemes r seen in the lesions too. Late secondary= organisms
can no longer be seen but the AB tests are still +. tertiary= look for FTA and
TPHA antibodies in CSF.
PCR is the best for confirmation of any diagnosis??

43
ID MERGED UNITY
February 17, 2016

Management
• benzylpenicillin
• alternatives: doxycycline
• the Jarisch-Herxheimer reaction is sometimes seen following
treatment. Fever, rash, tachycardia after first dose of antibiotic. It is
thought to be due to the release of endotoxins following bacterial
death and typically occurs within a few hours of treatment.




Q: 664 A 35yo IVDA on penicillin and flucloxacillin for cellulitis now presents with
jaundice, pale stools
and dark urine. What is the single most likely dx?
a. Hep A
b. Cholestatic jaundice
c. Chronic active hepatitis
d. Primary biliary cirrhosis
e. Hep B


Clincher(s) IV drug abuser, on penicillin and flucoxacillin forn cellulits, develops
obstructive jaundice.
A
B
C
D
E
KEY b. Cholestatic jaundice

Additional This iv drug abuser has developed drug-induced cholestatic/obstructive
Information jaundice.
Other drugs that can cause choloestatsis are… fusidic acid, co-amoxiclave,
nitrofurantoin, steroids, sulfonylureas, prochloperazine and chlorpromazine.
Reference Ohcm 251
Dr Khalid/Rabia Cholestatic jaundice associated with flucloxacillin therapy.
Although it's a vague scenario, the diagnosis of drug induced hepatic injury (
DIHI ) requires these attributes 1) history of drug exposure 2) exclusion of
other causes of hepatic injury , like hepatitis etc 3) improvement after stopping
the suspected drug 4) exacerbation/recurrence after the use of offending drug
Pale stool = cholestasis



Q: 680 A 35yo man has a temp=39C, cough with purulent sputum and right sided

44
ID MERGED UNITY
February 17, 2016

chest pain on
inspiration. He has herpes labialis. What is the single most likely causative
organism?
a. Coagulase +ve cocci in sputum
b. Gram -ve diplococci in sputum
c. Gram +ve diplococci in sputum
d. Pneumocystis carinii in sputum
e. Serology for legionella


Clincher(s) High temperature, cough, purulent sputum, right sided chest pain on
inspiration, herpes labialis.
A staphs
B
C
D AIDS
E Not required
KEY c. Gram +ve diplococci in sputum

Additional Community acquired pnemococcal pneumonia….strep pnemonie most imp
Information and common causative agent
Reference
Dr Khalid/Rabia Pneumococcus is the most common , and presents with herpes labialis apart
from other symptoms
Herpes labialis is a benign viral infection. If this is associated with a
pneumonia, it is usually a pneumococcal pneumonia. It is a self limiting
condition that RESOLVES in 10-15 days.
which is Streptococcus pneumoniae (Pneumococcus) which is coagulase &
catalase (-)ve, alpha hemolytic Gram (+)ve diplococci (Inulin, bile fermentation
& optochin sensitivity distinguishes it from Viridans group which is quite
similar and is out of the scope of the question).

The key to answer is again "statistical" one (i.e the most common cause of
community-acquired pneumonia in this age group).

The herpes labialis is often associated with streptococcal pneumonia which is
another key feature.

Pneumocystis carinii would cause a noticeable dyspnea in an
immunosuppressed individual or known case of HIV with CD4 count less than
200.

Legionella would cause diarrhea, altered level of consciousness & SIADH
(hyponatremia) in an elderly.

Coagulase (+)ve is staphylococcal group in which S.aureus genus is responsible

45
ID MERGED UNITY
February 17, 2016

for a pneumonia following influenza in a debilitated elderly patient which is
multilobular, pneumatocele, abscesses and necrotizing with dissemination,
endocarditis and osteomyelitis.



Q: 699 A 6yo child presented with drooling of saliva and severe stridor. He is febrile
and sick looking. XR Neck in extension shows a thumb sign. Choose the single
most likely dx.
a. Croup
b. URTI
c. Diphtheria
d. Acute epiglottitis


Clincher(s) 6 year old with stridor and drooling salive, febrile and sick looking. Xray neck
shows thumb sign in extension.
A
B
C
D
E
KEY d. acute epiglottitis
Additional The thumb sign is a manifestation of an oedematous and enlarged epiglottis
Information which is seen on lateral soft-tissue radiograph of the neck, and it suggests a
diagnosis of acute infectious epiglottitis.
Epiglottitis is a medical emergency caused by inflammation of
the epiglottis andaryepiglottic folds 1.

It is a life threatening condition which can lead to acute airway obstruction.


Hence, treatment should be urgent and performed by appropriately trained
individuals, e.g. instrumentation of the trachea should be performed by a
senior or a trained anaesthetist.

The commonest age of presentation is in children of 3 to 6 years. It is


exceedingly rare in the adult population.

Epiglottitis is almost always caused by infection Haemophilus influenzae type


B (HIB). However, with the increasing availability of the HIB vaccine, the
incidence of epiglottitis has changed. Indeed, many countries how have a
vaccination programme that prophylactically treats all children, e.g. UK.

Clinical presentation
Presentation generally has a very rapid course beginning with sore throat and
leading to difficulty swallowing and drooling.However, other presenting
symptoms include:

46
ID MERGED UNITY
February 17, 2016

• fever
• difficulty speaking
• muffling or changes in the voice
• inspiratory stridor
• severe dysphagia
Radiographic features
Radiograph
Lateral radiograph demonstrates thickening of epiglottis and aryepiglottic folds
which is called thumb sign . This is the radiographic corollary of the omega
sign. Hypopharynx may be over-distended.


Also the other signs and symptoms points towards acute epiglotitis
Reference Radiolopeadia.org
Dr Khalid/Rabia most common in children between the ages of 2 and 8

The most common symptoms


• Sore throat.
• Odynophagia (painful swallowing).
• Drooling (inability to swallow secretions).
• Fever.
• Anterior neck tenderness over the hyoid bone.

Fibre-optic laryngoscopy remains the 'gold standard' for diagnosing epiglottitis


radiograph of the neck may show the 'thumbprint sign'.




Q: 705 A 3yo boy has a sudden onset of fever, vomiting and bilateral face swelling.
Few days earlier the GP saw him for bilateral parotid pain and gave analgesics.
What is the most appropriate next step?
a. Analgesic
b. Antibiotic
c. Biopsy
d. Immediate surgery
e. Reassurance


Clincher(s)
A Already prescribed by gp
B Not required
C Not required

47
ID MERGED UNITY
February 17, 2016

D Not required
E
KEY
Additional Treatment for mumps is focused on relieving symptoms until your body’s
Information immune system fights off the infection. There are currently no medications
to treat the mumps virus.
The infection usually passes within a week or two.
In the meantime, the measures below may help.
• get plenty of bed rest until your symptoms have passed
• take over-the-counter painkillers, such as ibuprofen orparacetamol, to
relieve any pain (children aged 16 or under should not be given aspirin)
• drink plenty of fluids, but avoid acidic drinks such as fruit juice as these
can irritate your parotid glands; water is usually the best fluid to drink
• apply a warm or cool compress to your swollen glands to help
reduce any pain
• eat foods that don't require a lot of chewing, such as soup, mashed
potatoes and scrambled eggs
If your symptoms don’t improve after seven days, or suddenly worsen, contact
your GP for advice.

Reference nhs
Dr Khalid/Rabia This is a case of mumps. It can be unilateral or bilateral. Can cause distortion of
the face due to swelling. Pain is usually around the angle of the jaw. Fever can
be as high as 39.5 and it can cause dryness of the mouth due to blockage of
the salivary ducts. No specific treatment is required. Just keep the patient
hydrated and treat symptomatically.


Q: 725 A young man develops itching worse at night and following bathing. Exam:
greysish white linear rash can be seen on the wrist and periumbilical area.
What is the dx?
a. Scabies
b. Polycythemia
c. Urticarial
d. Atopic eczema
e. Lichen planus


Clincher(s) Itching worse at night following bath. Greyish white linear rash on wrists and
periumblical area
A
B
C

48
ID MERGED UNITY
February 17, 2016

D
E Non-infectious itchy rash
KEY
Additional
Information
Reference
Dr Khalid/Rabia Widespread itching, worse at night and when the person is warm. Skin signs
can vary.



Q: 756 A woman presents with hx of falls, becomes pale and clumsy. She is
hypertensive and takes atenolol, bendroflumethiazide and amlodipine. What
inv is needed?
a. 24h ECG
b. 24h BP monitoring
c. ECG
d. Echo
e. CT head


Clincher(s) Antihypertensive medication and history of falls.
A
B
C
D
E
KEY 24 hour bp monotoring
Additional Postural hypotension due to antihypertensice
Information medication….bendrofluthiazide
Reference
Dr Khalid/Rabia


Q: 809 A 66yo male presents with painful swallowing. What is the most likely dx?
a. Neisseria meningitides
b. Cryptococcus neoformans
c. Candida albicans
d. Isospora belli
e. Mycobacterium avium



Clincher(s) 66 year old presents with painlful swallowing.
A

49
ID MERGED UNITY
February 17, 2016

B
C
D
E
KEY C
Additional
Information
Reference
Dr Khalid/Rabia Candida infections are more common in old age, diabetics and in
immunocompromised

Isospora belli causes an intestinal infection mostly in HIV positive patients.

MAC can cause fevers, diarrhea, malabsorption, as well as loss of appetite and
weight loss, and can disseminate to the bone marrow

Cryptococcus mainly cause fungal meningitis and encephalitis in
immunocompromised and HIV positive.

Oesophageal candidiasis

Presentation
• Dysphagia, pain on on swallowing food or fluids and/or
retrosternal pain, usually with oropharyngeal candidiasis.
• This combination of symptoms is predictive of oesophageal
candidiasis.
• It is most often associated with treatment of haematopoietic or
lymphatic malignancies.
• In HIV-positive patients, it is an AIDS-defining illness.

Diagnosis
• A therapeutic trial of fluconazole for patients is useful; most
patients will respond within 7 days of treatment.
• Definitive diagnosis is by endoscopy.

Management of oesophageal candidiasis


• Consider admission to hospital - oesophageal candidiasis is a life-
threatening infection.
• Treat for 14-21 days; the following treatment options are
suggested:
o First-line treatment options:
§ Oral fluconazole (200-400 mg daily).

50
ID MERGED UNITY
February 17, 2016

Intravenous (IV) fluconazole, an
§
echinocandin - eg, caspofungin, or
amphotericin.[10]
o Second-line drugs are oral itraconazole oral solution, IV
posaconazole or IV voriconazole.
• For AIDS patients, HAART is advised to prevent recurrence.




Q:147 A 6yo pt comes with easy bruising in different places when she falls. CBC:
WBC=25, Hgb=10.9, Plt =45. Her paul brunnel test +ve. What is the most likely
dx?
a. Glandular fever
b. ITP
c. Trauma
d. NAI
e. Septicemia


Clincher(s) Paul brunnel test +
A
B
C
D
E
KEY A
Additional – glandular fever is often referred to as the "kissing disease"
Information Incubation period is 4-5 wks. Lymph nodes enlarge on the posterior triangle
area. Tx is usually supportive such as paracetamol , fluid and rest.
Steroid and acyclovir can be helpful in severe cases.
Paul Brunel test or heterophil antibody test positive suggestive of infectious
mononucleosis
Reference
Dr Khalid/Rabia Suggestive lab. Values: WBC=25 (leucocytosis), Hgb=10.9 (usually patient is not
anaemic), Plt=45 (thrombocytopenia-leading to easy bruising), Positive paul
bunnel test.


Q:176 A 23yo lady presents with headache. Exam: photophobia and generalized rash
that doesn’t blanch on pressure. What must be done immediately?
a.IV benzylpenicillin
b . Isolate pt
c. Gown and mask
d. Blood culture

51
ID MERGED UNITY
February 17, 2016


Clincher(s) Rash which doesn't blanch on pressure, photophobia and headache.
A
B
C
D
E
KEY A
Additional IV benzylpenicillin should be given to all patients before transferring them to
Information hospital to avoid septicaemia.
Reference
Dr Khalid/Rabia


Q:340 A pt complains of SOB, wheeze, cough and nocturnal waking. He has dry scaly
shin with rashes that are itchy. What is the single most likely dx?
a. Scabies
b. Eczema
c. Rheumatism
d. Dermatitis
e. Psoriasis


Clincher(s) SOB, wheeze, cough, nocturnal waking with dry scaling skin with rashes that
are itchy showing atopic features.
A
B
C
D
E
KEY B
Additional
Information
Reference
Dr Khalid/Rabia


Q:408 A pt from Africa comes with nodular patch on the shin which is reddish brown.
What is the most probable dx?
a. Lupus vulgaris
b. Erythema nodosum
c. Pyoderma gangrenosum
d. Erythema marginatum
e. Solar keratosis

52
ID MERGED UNITY
February 17, 2016


Clincher(s) Pt from African ethnicity presented with nodular patch on the shin which are
reddish brown directing us towards sarcoidosis picture.
A It's a skin TB which presents as jelly like nodules.
B Present in sarcoidosis, TB, leprosy and various drugs
C Present in malignancy and IBD
D Present in rheumatic fever.
E Pre malignant condition to SCC. Yellowish scaly plaques.
KEY B
Additional
Information
Reference
Dr Khalid/Rabia Causes of erythema nodosum: MOST COMMON CAUSES- i) streptococcal
infection ii) sarcoidosis. Other causes- tuberculosis, mycoplasma pneumonia,
infectious mononucleosis, drugs- sulfa related drug, OCP, oestrogen; Behcets͛
disease, CD, UC; lymphoma, leukemia and some others].


Q:562 A 34yo female presents with a foul smelling discharge. What set of organisms
are we looking for to be treated here?
a. Chlamydia, gonorrhea
b. Chlamydia, gardenella
c. Chlamydia, gonorrhea, gardenella
d. Gonorrhea, gardenella
e. Gardenella only


Clincher(s)
A
B
C
D
E
KEY E
Additional Trichomoniasis is a common STI caused by a tiny parasite. It can make your
Information vaginal discharge frothy, yellow or green. You may have a lot of discharge,
which may also have an unpleasant fishy smell

Reference
Dr Khalid/Rabia Gardenella only. [Here fowl smelling discharge is caused by gardenella and
gonorrhea. Gardenella being most common we shall treat it first and if it
doesn’t work we shall go for gonorrhea work up also]. Dr Khalid

Dr Rabia

53
ID MERGED UNITY
February 17, 2016

Chlamydia is usually asymptomatic (no odour) and generally goes with
gonorrhea. BV will give the grey fish-smelling discharge
Bacterial vaginosis and Trichomonas vaginalis give foul smelling discharge. In
BV its grey white fishy and in TV it can be greenish frothy fihy alongwith
vulvovginitis i-e strawberry cervix. The discharge of Chlamydia
and Gonorrhea is not foul smelling but gives dysuria.

Bacterial vaginosis (BV) describes an overgrowth of predominately anaerobic
organisms such as Gardnerella vaginalis.
Amsel's criteria for diagnosis of BV - 3 of the following 4 points should be
present
· thin, white homogenous discharge
· clue cells on microscopy: stippled vaginal epithelial cells
· vaginal pH > 4.5
· positive whiff test (addition of potassium hydroxide results in fishy odour)

Management
· oral metronidazole for 5-7 days



Q:567 A 78yo nursing home resident is revived due to the development of an
intensely
itchy rash. Exam: white linear lesions are seen on the wrists and elbows and
red papules are present on the penis. What is the most appropriate
management?
a. Topical permethrin
b. Referral to GUM clinic
c. Topical betnovate
d. Topical ketoconazole
e. Topical selenium sulfide hyoscine


Clincher(s) Elderly nursing home resident with itchy rash and white linear lesions on
wrist and elbow and red apples on the penis.
A
B
C
D
E
KEY A
Additional
Information
Reference
Dr Khalid/Rabia Red papule on penis typical with wrist and elbow lesion goes with Scabies,
topical permethrin once wk and repeat if symptoms remain. white linear

54
ID MERGED UNITY
February 17, 2016

lesions


Q:569 9. A pt presents with gradual onset of headache, neck stiffness, photophobia
and fluctuating LOC. CSF shows lymphocytosis but no organism on gram stain.
CT brain is normal. What is the single most likely dx?
a. Hairy leukoplakia
b. TB
c. CMV infection
d. Candida infection
e. Cryptococcal infection

Clincher(s) Gradual onset of headache, neck stiffness, photophobia, fluctuating loc.
Lymphocytosis but no organism on gram stain and CT was normal.
A Associated with EBV and HIV
B
C Present with eye symptoms, pneumonitis, colitis, hepatitis.
D
E This causes chronic meningitis with no neck stiffness. Tx is Amphotericin B IV +
flucytosine.
KEY B
Additional
Information
Reference
Dr Khalid/Rabia classic triad of fever, neck stiffness and a change in mental status was present
in only 44% of adults presenting with community-acquired acute bacterial
meningitis. However, 95% had at least two of the four symptoms of headache,
fever, neck stiffness and altered mental status.
Most patients with viral meningitis present with subacute neurological
symptoms developing over 1-7 days. Chronic symptoms lasting longer than
one week suggest meningitis caused by some viruses as well as TB, syphilis or
fungi.






Q:572 A pt presents with fever, dry cough and breathlessness. He is tachypneic but
chest is clear. Oxygen saturation is normal at rest but drops on exercise. What
is the single most likely dx?
a. CMV infection
b. Candida infection
c. Pneumocystis carinii infection
d. Cryptococcal infection
e. Toxoplasma abscess

55
ID MERGED UNITY
February 17, 2016



Clincher(s) Fever, dry cough, sob, tachpnoea but chest is clear. Oxygen saturation is
normal but drops on exercise.
A
B
C
D
E
KEY C
Additional CXray normally clear and in CT scan diffuse ground glass appearance can be
Information seen. Tx is co trimoxazole.
Reference
Dr Khalid/Rabia This kind of history about oxygen desaturation on exercise is typical for PCP.
Dry cough along with that supports that.
patient is immunocompromised. Mostly in HIV patients we see that pt
becomes breathless after a walk or exercise. Also fever with dry cough is there.
Most likely pathogen is PCP.
CMV affects retina n brain in immunocompromised and post transplant pts.
Toxoplasmosis..brain, eye and muscle. Usually one eye involvement.
Candida. .mouth and esophagus
Cryptococcus..meningitis

Whilst the organism Pneumocystis carinii is now referred to as Pneumocystis
jiroveci, the term Pneumocystis carinii pneumonia (PCP) is still in common use


Q:577 A 3m baby was miserable and cried for 2h following his 1st routine
immunization
with DTP, HiB and meningitis. What is the single most appropriate action?
a. Defer immunization for 2wks
b. Don’t give vaccine
c. Give half dose of vaccine
d. Give paracetamol with future doses of the same vaccine
e. Proceed with standard immunization schedule


Clincher(s) Baby cried after the first vaccination
A In febrile illness.
B If there's any CI
C
D
E
KEY E
Additional

56
ID MERGED UNITY
February 17, 2016

Information
Reference
Dr Khalid/Rabia

Q: A 12yo boy with T1DM has poor long-term control. He is unconscious,
hyperventilating and
dehydrated. His blood glucose is 28mmol/l. What is the single most imp initial
tx?
a. Albumin IV
b. Bicarbonate IV
c. Insulin IV
d. Insulin SC
e. Saline 0.9% IV



Clincher(s) Case of DKA
A
B
C
D
E
KEY e. Saline 0.9% IV
Management
• fluid replacement: most patients with DKA are deplete around 5-8
litres. Isotonic saline is used initially
• insulin: an intravenous infusion should be started at 0.1 unit/kg/hour.
Once blood glucose is < 15 mmol/l an infusion of 5% dextrose should be
started
correction of hypokalaemia
Additional
Information
Reference
Dr Khalid/Rabia


Q: 1298 A man developed intense pain after using the end of a pencil to scratch his
inner ear. He took out the pencil from his ear and realized the end of the pencil
with the rubber part is still stuck in his ear. What is the most appropriate
management?
a. Remove with a hook
b. Instill olive oil
c. Remove GA
d. Remove with magnet instrument
e. Do syringing

57
ID MERGED UNITY
February 17, 2016



Clincher(s)
A
B
C
D
E Usually done for wax removal. Irrigation is often effective. Irrigation with
water is contra-indicated for soft objects, organic matter or seeds (which may
swell and increase the level of pain and difficulty to remove if exposed to
water
KEY a. Remove with a hook
Additional
Information • Insects should be killed prior to removal, using 2% lidocaine.
• Remove batteries or magnets as soon as possible to prevent corrosion
orburns. Do not crush a battery during removal.
• Adhesives (eg, Super Glue®) may be removed manually within 1-2 days
once desquamation has occurred.

Methods for removal


• Forceps or hook: grasp the object with forceps, or place a hook behind
the object and pull it out.
• Irrigation is often effective. Irrigation with water is contra-indicated for
soft objects, organic matter or seeds (which may swell and increase the
level of pain and difficulty to remove if exposed to water).
• Suction with a small catheter held in contact with the object may be
effective.

Reference
Dr Khalid/Rabia


Q:1301 The biological parents of a child with CF come to you to know about the
chances of their future
children with the same disease. What would you say to them?
a. There is a 1:4 chance that your future child will have this disease
b. All their unaffected children will be carriers of CF
c. Nothing can be predicted
d. It can 100% dx antenatally


Clincher(s)
A When two people who carry the cystic fibrosis gene have a child, there is a:
1 in 4 chance that the child will have cystic fibrosis (by inheriting the cystic

58
ID MERGED UNITY
February 17, 2016

fibrosis gene from both parents
B 2 in 4 chance that the child will not have cystic fibrosis but will be a carrier (by
inheriting a cystic fibrosis gene from one parent but the normal gene from the
other parent).


C
D
E
KEY A – 1 in 4
Additional
Information
Reference
Dr Khalid/Rabia Cystic Fibrosis is an autosomal recessive disorder.
1 in 4 chance that the child will not have cystic fibrosis and will not be a
carrier (by inheriting the normal gene from both parents).




Q:1321 Which of the following conditions requires operative management?
a. Cellulitis
b. Dyshidrosis
c. Erysipelas
d. Fournier’s gangrene
e. Lymphangitis


Clincher(s)
A Cellulitis is an infection of the deep layer of skin (dermis) and the layer of fat
and tissues just under the skin (the subcutaneous tissues.

B This is a dermatitis or eczema of unknown aetiology, characterised by an itchy


vesicular eruption of the hands, fingers and soles of the feet. It can be acute,
recurrent or chronic and is difficult to treat effectively. It is more common in
spring and summer and in countries with warmer climates.
C Erysipelas is an infection of the skin which is nearer to the skin surface (more
superficial) than cellulitis
D
E Infection of lymph vessel
KEY Fournier’s gangrene is a medical emergency that is difficult to recognise in the
early stages

It can be classified according to the causative organism:

59
ID MERGED UNITY
February 17, 2016


• type 1 is caused by mixed anaerobes and aerobes (often occurs post-
surgery in diabetics)
• type 2 is caused by Streptococcus pyogenes

Features

• acute onset
• painful, erythematous lesion develops
• extremely tender over infected tissue

Management

• urgent surgical referral debridement
• intravenous antibiotics

Additional Dihydrosis – patient.info
Information
• The affected areas are the centre of the palms or soles.
• It is usually symmetrical.
• After several hours of itching or burning in the hands, feet or both, the
eruption develops. Tiny vesicles, about 1 or 2 mm in diameter, erupt, first
along the lateral aspects of the fingers and then on the palms or soles.
• Palms and soles may be red and wet with perspiration.

The condition is self-limiting but, as it can be intensely itchy, symptomatic


treatment may be in order. In practice, many patients require a combination of
treatments:[11] [12]

• Burow's solution (10% aluminum acetate in a 1:40 dilution), is a drying


soak that can be used if the lesions ooze.
• Large blisters can be drained under aseptic conditions.
• Antibiotics are only required if secondary infection occurs.
• Strong topical steroids to control itching, and cold compresses (eg, a
1:10.000 solution of potassium permanganate), are the usual first-line
treatment.


Reference
Dr Khalid/Rabia

60
ID MERGED UNITY
February 17, 2016


Q:1323 A house-bound 78yo man with severe COPD has had a gradual deterioration
over recent months and is now breathless at rest. He is on maximal inhaled
medical therapy. Result: pH=7.36, PaCO2=5.9kPa, PaO2=6.9kPa. What is the
single most appropriate additional tx?
a. Aminophylline PO
b. ACEi PO
c. Antibiotic PO
d. Oxygen
e. Steroid PO


Clincher(s)
A
B
C
D
E
KEY D-OXYGEN
Additional
Information
Reference
Dr Khalid/Rabia The 2010 NICE guidelines on COPD clearly define which patients should be
assessed for and offered long-term oxygen therapy (LTOT). Patients who
receive LTOT should breathe supplementary oxygen for at least 15 hours a day.
Oxygen concentrators are used to provide a fixed supply for LTOT.

Assess patients if any of the following:

• very severe airflow obstruction (FEV1 < 30% predicted). Assessment
should be 'considered' for patients with severe airflow obstruction
(FEV1 30-49% predicted)
• cyanosis
• polycythaemia
• peripheral oedema
• raised jugular venous pressure
• oxygen saturations less than or equal to 92% on room air

Assessment is done by measuring arterial blood gases on 2 occasions at least 3
weeks apart in patients with stable COPD on optimal management.

Offer LTOT to patients with a pCO2 of < 7.3 kPa or to those with a pO2 of 7.3 -
8 kPa and one of the following:

• secondary polycythaemia
• nocturnal hypoxaemia

61
ID MERGED UNITY
February 17, 2016

• peripheral oedema
• pulmonary hypertension


Q:1324 A 79yo man has a swelling of the right groin which was clinically dx to be
indirect inguinal hernia. What is the single feature of the hearnia sac that
would confirm the dx?
a. Comes through femoral ring
b. Doesn’t pass through the deep inguinal ring
c. Lies below and lateral to the pubic tubercle
d. Only passes through the superficial inguinal ring
e. Passes through the deep inguinal ring


Clincher(s)
A
B
C
D
E
KEY Ans E

Additional There are two types of inguinal hernia, direct and indirect, which are defined
Information by their relationship to the inferior epigastric vessels. Direct inguinal hernias
occur medial to the inferior epigastric vessels when abdominal contents
herniate through a weak spot in the fascia of the posterior wall of the inguinal
canal, which is formed by the transversalis fascia. Indirect inguinal hernias
occur when abdominal contents protrude through the deep inguinal ring,
lateral to the inferior epigastric vessels; this may be caused by failure of
embryonic closure of theprocessus vaginalis..

Reference
Dr Khalid/Rabia


Q:1330 A young child dx with chicken pox. Usually goes to day care. What is the most
appropriate
advice?
a. Child should be admitted to hospital straight away
b. Isolate the child from parents and siblings at home
c. Advice that he can go back to nursery when the rash is crusted over


Clincher(s)
A

62
ID MERGED UNITY
February 17, 2016

B
C
D
E
KEY ANS IS C
Additional
Information Chickenpox is caused by primary infection with varicella zoster virus. Shingles
is reactivation of dormant virus in dorsal root ganglion

Chickenpox is highly infectious

• spread via the respiratory route
• can be caught from someone with shingles
• infectivity = 4 days before rash, until 5 days after the rash first
appeared*
• incubation period = 10-21 days

Clinical features (tend to be more severe in older children/adults)

• fever initially
• itchy, rash starting on head/trunk before spreading. Initially macular
then papular then vesicular
• systemic upset is usually mild

Management is supportive

• keep cool, trim nails
• calamine lotion
• school exclusion: current HPA advice is 5 days from start of skin
eruption. They also state 'Traditionally children have been excluded
until all lesions are crusted. However, transmission has never been
reported beyond the fifth day of the rash.'
• immunocompromised patients and newborns with peripartum
exposure should receive varicella zoster immunoglobulin (VZIG). If
chickenpox develops then IV aciclovir should be considered

A common complication is secondary bacterial infection of the lesions. Rare
complications include

• pneumonia
• encephalitis (cerebellar involvement may be seen)
• disseminated haemorrhagic chickenpox
• arthritis, nephritis and pancreatitis may very rarely be seen

Reference
Dr Khalid/Rabia

63
ID MERGED UNITY
February 17, 2016



Q:1337 . A man suffers from Herpes Zoster affecting his face. Which of the following
mucos membrane is
to be affected?
a. Cheek
b. Cornea
c. Conjunctiva
d. Oropharynx
e. Palate


Clincher(s)
A
B
C
D
E
KEY This one is controversial. Tried to find out on Facebook PLAB FORUM. Most of
the people agreed on conjunctiva as cornea is not a mucous membrane
Additional
Information
Reference
Dr Khalid/Rabia


Q:1342 A 16yo boy in boarding school feels unwell. He developed cough and rash .His
CXR showed bilateral consolidations. What is the cause of his symptoms?
a. Staph aureus
b. Legionella
c. Mycoplasma
d. Streptococcus


Clincher(s)
A
B In legionella questions u ll mostly find some water related hints or outdoor
activity near water or air conditioning system mention and most of the times
GI symptoms along with respiratory symptoms

C Homeless shelters ,Young army recruits living in barracks ,young students in
dormitories(in boarding schools) ------->mycoplasma,

D
E

64
ID MERGED UNITY
February 17, 2016

KEY
Additional Mycoplasma pneumoniaeis a cause of atypical pneumonia which often affects
Information younger patients. It is associated with a number of characteristic complications
such as erythema multiforme and cold autoimmune haemolytic anaemia.
Epidemics ofMycoplasma pneumoniaeclassically occur every 4 years. It is
important to recognise atypical pneumonias as they may not respond to
penicillins or cephalosporins due to it lacking a peptidoglycan cell wall.

Features

• the disease typically has a prolonged and gradual onset
• flu-like symptoms classically precede a dry cough
• bilateral consolidation on x-ray
• complications may occur as below

Complications

• cold agglutins (IgM) may cause an haemolytic anaemia,
thrombocytopenia
• erythema multiforme, erythema nodosum
• meningoencephalitis, Guillain-Barre syndrome
• bullous myringitis: painful vesicles on the tympanic membrane
• pericarditis/myocarditis
• gastrointestinal: hepatitis, pancreatitis
• renal: acute glomerulonephritis

Investigations

• diagnosis is generally by Mycoplasma serology
• positive cold agglutination test

Reference
Dr Khalid/Rabia

Q:1350 . A 29yo man took a tour of Japan and also travelled to other parts of Asia,
developed fever,
petecia and rash on his body. He didn’t take malaria prophylaxis prior to travel.
What is the most
likely dx?
a. Malaria
b. HSP
c. HIV
d. Dengue fever
e. ITP
etc

65
ID MERGED UNITY
February 17, 2016


Clincher(s) Petechae due to low platelets and travel Hx to asia are cinchers here
A
B
C
D
E
KEY Petechae due to low platelets and travel Hx to asia are cinchers here. Dengue
fever is a viral infection which can progress to viral haemorrhagic fever (also
yellow fever, Lassa fever, Ebola
Additional )
Information
Basics

• transmitted by the Aedes aegyti mosquito
• incubation period of 7 days
• a form of disseminated intravascular coagulation (DIC) known as
dengue haemorrhagic fever (DHF) may develop. Around 20-30% of
these patients go on to develop dengue shock syndrome (DSS)

Features

• causes headache (often retro-orbital)
• fever
• myalgia
• pleuritic pain
• facial flushing (dengue)
• maculopapular rash

Treatment is entirely symptomatic e.g. fluid resuscitation, blood transfusion
Reference
Dr Khalid/Rabia

Q:1353 A child has developed rash after the tx of penicillin. What will be the cause of
rash?
a. Drug reaction
b. Kawasaki
c. Inf Mono
Drug reaction
.


Clincher(s)
A
B

66
ID MERGED UNITY
February 17, 2016

C
D
E
KEY DRUG reaction
Additional Mild to moderate allergic reactions to penicillin are common, and symptoms
Information may include any of the following:
· Hives (raised, extremely itchy spots that come and go over a period of
hours)
· Tissue swelling under the skin, typically around the face (also known as
angioedema)
· Throat tightness.
· Wheezing.
· Coughing
Reference
Dr Khalid/Rabia

Q: 10 A 53yo female presents with an acute painful hot knee joint. She is a known
case of RA. Onexamination, the knee is red, tender and swollen. The hamstring
muscles are in spasm. Hertemp is 38.5C and BP is 120/80mmHg. What is the
SINGLE best next inv?
a. Joint aspiration for cytology and culture and sensitivity
b. Joint aspiration for positive birefrengent crystals
c. Joint aspiration for negative birefrengent crystals
d. Blood culture
e. Serum uric acid




Clincher(s)
A Any chronically arthritic joint is predisposed to infection. Moreover chronic use
of steroid in Rh. arthritis is one of the important predisposing factor.

What is the likely organism in younger age group?
Staphylococcus
Neisseria gonorrhoeae

B
C
D
E
KEY A. Joint aspiration for cytology and culture and sensitivity.
Additional
Information
Reference

67
ID MERGED UNITY
February 17, 2016

Dr Khalid/Rabia RA always involves bilateral symmetrical joints with morning stiffness. The
patient presented with new complaint which is monoarticular, swollen n hot.
It's clearly Septic arthritis n u do joint aspiration. Chronic use of steroids is
one of the important predisposing factors.

Diagnosis : Septic Arthritis due to persistent Rheumatoid Arthritis.
The classic picture is a single swollen joint with pain on active or passive
movement.
It is more common in patients with prior joint damage, as in gout,
rheumatoid arthritis and systemic connective tissue disorders.
Fever and rigors. Chest wall pains.
Treatment :
Flucloxacillin and for MRSA - Vancomycin.
Penicillin with Gentamicin is being used as well.



Q: 12 An 18yo female has periorbital blisters. Some of them are crusted, others
secreting pinkish fluid. What is the most likely dx?
a. Shingles
b. Chicken pox
c. Varicella
d. Rubella
e. Measles



Clincher(s)
A . Ophthalmic division of trigeminal nerve.
. Typically shingles is unilateral.

B
C
D
E
KEY A. Shingles
Additional
Information
Reference
Dr Khalid/Rabia Short note everywhere. Mcq covers it up.
Treatment :
Refer to ophthalmologist. Ocular lubricants, cool compressors, topical
steroids,
Botulinum toxin injection if neurotrophic ulcers form.

68
ID MERGED UNITY
February 17, 2016


Q: 18 A 7yo child is brought to the ED with a 1 day hx of being listless. On
examination, the child isdrowsy with an extensive non-blanching rash. What
advice would you give the parents?
a. All family members need antibiotic therapy
b. Only the mother should be given rifampicin prophylaxis
c. All family members need isolation
d. All family members should be given rifampicin prophylaxis


Clincher(s)
A
B
C
D
probable diagnosis Meningococcal disease.
diagnostic investigation Blood or CCF PCR
initial management Prehospital management: Benzyl penicillin or cefotaxime.

E
KEY D. All family member should be given rifampicin prophylaxis
Additional
Information
Reference
Dr Khalid/Rabia DIAGNOSIS : Meningococcal infection. It can be meningococcal Septicemia
which is without Meningitis, or can be Meningococcal Meningitis. This Mcq
doesnt have meningitis signs so we will stick to M.Septicemia.

Caused by N.Meningitidis
Signs and Symptoms :
Most common and important - Non - blanching rash
Fever, headache
May have : Stiff neck, back rigidity, bulging fontanelle (in infants),
photophobia.
Altered mental state, unconsciousness, toxic/moribund state, Kernig's sign
(pain and resistance on passive knee extension with hips fully flexed) and
Brudziñski's sign (hips flex on bending the head forward)

Pre-Hospital Management :
Call 999 and give Benzyl penicillin or Cefotaxime

INVESTIGATIONS :
• Blood cultures.
• FBC (WCC), CRP, U&Es, renal function tests, LFTs.
• Blood test for polymerase chain reaction (PCR): perform whole
blood real-time PCR testing - (EDTA sample) - for N. meningitidis

69
ID MERGED UNITY
February 17, 2016

to confirm a diagnosis of meningococcal disease
• Investigations for disseminated intravascular coagulation:
prothrombin time is elevated, activated partial thromboplastin
time (aPTT) is elevated, platelet count is reduced and the
fibrinogen level is low.
• Lumbar puncture - once the patient is stable
• Aspirate from other sterile sites suspected of being infected (eg,
joints) for microscopy, culture and PCR.

TREATMENT :

• Choice of antibiotics in hospital :


o Ceftriaxone is usually given to those over 3 months
o Cefotaxime and amoxicillin are usually given to those
under 3 months.
o Vancomycin is given in addition, to those who have
recently travelled outside the UK or have had
prolonged or multiple exposure to antibiotics.
• CHEMOPROPHYLAXIS :
• To close contacts of cases, irrespective of vaccination status - for
example, those who have had prolonged close contact with the case
in a household-type setting during the seven days before onset of
illness
• Ciprofloxacin and rifampicin are both recommended by Public Health
England (PHE) but ciprofloxacin is the preferred choice for most
individuals.
• Ciprofloxacin can be used in all ages and in pregnancy; it is easily
available in a single dose and does not interfere with oral
contraceptives (but is contra-indicated if there has been previous
sensitivity):
o Adults and children aged >12 years - 500 mg orally stat.
o Children aged 5-12 years - 250 mg orally stat.
o Children aged <5 years - 30 mg/kg up to a maximum of 125 mg
orally stat.
• Rifampicin is no longer the drug of choice as, although it is licensed for
chemoprophylaxis, it has several disadvantages including important
drug interactions




Q: 19 A 47yo man has a temp of 39C and is delirious. He has developed blisters
mainly on his trunk,which appeared a few hours ago. He is well and not on any
medications. He last travelled 5months ago to Italy. Which of the following is
the most likely dx?
a. Shingles

70
ID MERGED UNITY
February 17, 2016

b. Chicken pox
c. Pemphigoid
d. Bullous pemphigus


Clincher(s) Blisters on trunk otherwise well.
A Involves specific dermatome
B
C They must mean pen phobia which is blisters in younger ppl.
D Bullous pemphigoid is a blistering skin disease that tends to affect the elderly.

E
KEY B
Additional Q 1. What is the likely key?
Information Q. 2. Why delirium in this patient?
Q. Is his travel history has any link to the development of this disease?
Ans. 1. B. Chicken pox
Ans. 2. Adults more commonly develop a more generalized brain
inflammation ("encephalitis") whose symptoms may include delirium and
seizures.
Ans. 3. Incubation period of chicken-pox is 10-21 days. So this travel history is
not significant.


Reference
Dr Khalid/Rabia


Q: 25 A resident of a nursing home presented with rashes in his finger webs and also
on his abdomen, with complaints of itching which is severe at night. He was dx
with scabies. What the best tx for
his condition?
a. 0.5% permethrin
b. Doxycycline
c. 5% permethrin
d. Reassure
e. Acyclovir
Q. 1. What is the likely key?
Q. 2. Will you consider any other treatment beside this?
Ans. 1. C
Ans. 2. Scabies outbreaks in nursing homes and cases of crusted scabies may
require combination therapy consisting of topical application of permethrin
and 2 oral doses of ivermectin at 200 mcg/kg (administered 1 wk apart). X

DIAGNOSIS : SCABIES

71
ID MERGED UNITY
February 17, 2016


• signs and symptoms develop after 3-4 weeks. Symptoms reappear
within 1-3 days if the person is re-infested due to prior
sensitisation.
• The most common presenting symptom is widespread itching. This
is usually worse at night and when the person is warm. A history of
several family members all suffering with itch is strongly suggestive
of scabies. Scratching predisposes to secondary bacterial infection.
• Lesions may be papules, vesicles, pustules, and nodules.
Erythematous papular or vesicular lesions are usually seen in the
sites of the burrows. The more widespread, symmetrical, itchy,
papular eruption is not in the areas of burrows or obvious mite
activity. This is most commonly seen around the axillae, the peri-
areolar region of the breasts in women, and the abdomen,
buttocks, and thighs.
• The 'wake' sign is specific for scabies, can be seen with the naked
eye and points towards the location of the mite.
• Hyperkeratotic crusted lesions called as Crusted Norwegian Scabies

INVS :
Ink Burrow Test.
Diagnosis can be confirmed by taking a skin scraping from an affected area.

TREATMENT :
First line : Permethrin 5%
2nd line : malathion 0.5% aqueous liquid


Clincher(s)
A
B
C
D
E
KEY C
Additional
Information
Reference
Dr Khalid/Rabia


Q: 32 A 33yo man presents with an itchy scaly annular rash on his thigh after a walk
in the park. Which of the following drugs will treat his condition?
a. Erythromycin
b. Doxycycline
c. Penicillin

72
ID MERGED UNITY
February 17, 2016

d. Amoxicillin


Clincher(s) Itchy scaly rash on thigh while walking in the park.
A
B
C
D
E
KEY B
Additional
Information
Reference
Dr Khalid/Rabia Q. 1. What is the key?
Q. 2. What is the diagnosis?
Ans. 1. b.
Ans. 2. Lyme disease.

Lyme disease is caused by the spirochaete Borrelia burgdorferi and is spread by
ticks

Features
• early: erythema chronicum migrans + systemic features (fever,
arthralgia)
• CVS: heart block, myocarditis
• neuro: cranial nerve palsies, meningitis


Investigation
• serology: antibodies to Borrelia burgdorferi


Management
• doxycycline if early disease. Amoxicillin is an alternative if doxycycline is
contraindicated (e.g. pregnancy)
• ceftriaxone if disseminated disease
• Jarisch-Herxheimer reaction is sometimes seen after initiating therapy:
fever, rash, tachycardia after first dose of antibiotic (more commonly
seen in syphilis, another spirochaetal disease)


Q: 50 A 22yo lady has been unwell for some time. She came to the hospital with
complaints of fever and painful vesicles in her left hear. What is the most
probable dx?
a. Acne
b. Herpes zoster

73
ID MERGED UNITY
February 17, 2016

c. Chicken pox
d. Insect bite
e. Cellulitis




Clincher(s) Fever and painful vesicles in the ear
A
B
C
D
E
KEY B
Additional
Information
Reference
Dr Khalid/Rabia . 1. What is the key?
Q. 2. What is the specific name of the condition?
Ans. 1. Herpes Zoster
Ans. 2. Herpes zoster oticus/Ramsay hunt syndrome.

Ramsey hunt syndrome
Painful vesicles in her left ear
• Features auricular pain is often the first feature
• facial nerve palsy
• vesicular rash around the ear
• other features include vertigo and tinnitus


Management
• oral aciclovir and corticosteroids are usually given




Q: 81 A 10yo girl presents with hoarseness of the voice. She is a known case of

bronchial asthma and has been on oral steroids for a while. What is the most

likely cause of hoarseness?

a. Laryngeal candidiasis

b. Infective tonsillitis

74
ID MERGED UNITY
February 17, 2016

c. Laryngeal edema

d. Allergic drug reaction

e. Ludwigs angina



Clincher(s)
A
B
C
D
E udwig's angina, otherwise known as angina ludovici, is a serious, potentially
life-threatening cellulitis,[1] or connective tissue infection, of the floor of the
mouth, usually occurring in adults with concomitant dental infections and if
left untreated, may obstruct the airways, necessitating tracheotomy.
KEY A
Additional
Information
Reference
Dr Khalid/Rabia Hoarseness may be a feature of larynge may occur in:


Infections - acute epiglottitis, diphtheria, croup, laryngeal abscess, laryngitis

Inflammation/oedema - airway burns, anaphylaxis, physical trauma, angio-

oedema, hereditary angio-oedema.

Vocal cord immobility - laryngeal nerve palsy (depending on the position of the

cords) or cricoarytenoid joint disease.

Immuno compromised states lead to fungal infections. These include HIV,

Diabetes etc. Patients taking long term steriods (inhalar or oral), antibiotics n

those having vitb12 and folic acid deficiecy are at a risk of having oral thrush.

75
ID MERGED UNITY
February 17, 2016

Q. 1. What is the key?

Q. 2. What is the reason for this condition?

Ans. 1. The key is A. Laryngeal candidiasis.

Ans. 2. Steroids predisposes to fungal infection.al obstruction - so can be a

warning of impending airway obstruction.




Q: 85 A man presented with cellulitis and swelling. He was started on flucloxacillin.

What other medication do you want to add?

a. Vancomycin

b. Penicillin

c. Metronidazole

d. Ceftriaxone

e. Amoxicillin


Clincher(s)
A
B
C
D
E
KEY B
Additional
Information
Ans. 1. The key is B. Penicillin.


Reference
Dr Khalid/Rabia

76
ID MERGED UNITY
February 17, 2016


Q: 86 A 24yo college student presents with nausea, vomiting, headache, neck

stiffness and a fever of 38.4C. What is the most appropriate empirical

antibiotic to be started?

a. Ceftriaxone

b. Penicillin

c. Gentamicin

d. Tazobactam

e. Meropenem



Clincher(s)
A A
B
C
D
E
KEY
Additional
Information
Reference
Dr Khalid/Rabia Ans. The key is A. Ceftriaxone. [In OHCM-Cefotaxime <55yrs and Cefotaxime +

Ampicillin if age >55yrs].

One should start benzyl penicillin before admission. After admission, ideally

cefotaxime should be given as per new guidelines. But, as there was no option

of cefotaxime, we would go for ceftriaxone (also 3rd gen cephalosporin)

77
ID MERGED UNITY
February 17, 2016


Q: 105 A 73yo male presents with enlarged cervical nodes. He has had recurrent
infections over the last year. His conjunctiva is pale. Choose the single cell type
you will find on the blood film.
a. Granulocyte without blast cells
b. Myelofibroblasts
c. Plasma cells
d. Mature lymphocytes


Clincher(s)
A
B
C
D
E
KEY D
Additional
Information
Reference
Dr Khalid/Rabia . 1. What is the key?
Q. 2. What is the diagnosis?
Q. 3. What are the points in favour of your diagnosis?
Ans. 1. The key is D. Mature lymphocytes.
Ans. 2. The diagnosis is CLL.
Ans. 3. It is CLL because of his age (73 yrs). Other supportive features are
cervical lymphadenpathy, recurrent infections (mature but functionally
defective lymphocytes), and pale conjunctiva (anaemia).

It is a typical presentation of CLL with reurrent infections, symmetrical
lymphadenopathy, anemia. There can also be hepatosplenomegaly and
thrombocytopenia leading to patechae
On blood film there will be B cell lymphocytosis often with smudge cells. There
are mature but functionally impaired lymphocytes as they escape apoptosis. In
bone marrow there is lymphocytic replacement of bone marrow cells.
Mainstay of treatment is chemotherapy.



Q:

Clincher(s)
A
B
C

78
ID MERGED UNITY
February 17, 2016

D
E
KEY
Additional
Information
Reference
Dr Khalid/Rabia


Q:

Clincher(s)
A
B
C
D
E
KEY
Additional
Information
Reference
Dr Khalid/Rabia


Q:

Clincher(s)
A
B
C
D
E
KEY
Additional
Information
Reference
Dr Khalid/Rabia

Q: 81 A 10yo girl presents with hoarseness of the voice. She is a known case of
bronchial asthma and has been on oral steroids for a while. What is the most
likely cause of hoarseness?
a. Laryngeal candidiasis
b. Infective tonsillitis
c. Laryngeal edema
d. Allergic drug reaction

79
ID MERGED UNITY
February 17, 2016

e. Ludwigs angina



Clincher(s)
A Steroids predisposes to fungal infection.
B
C
D
E
KEY The key is A. Laryngeal candidiasis.
Additional (Rabia) Hoarseness may be a feature of laryngeal obstruction - so can be a
Information warning of impending airway obstruction.

This may occur in:
Infections - acute epiglottitis, diphtheria, croup, laryngeal abscess, laryngitis
Inflammation/oedema - airway burns, anaphylaxis, physical trauma, angio-
oedema, hereditary angio-oedema.

Vocal cord immobility - laryngeal nerve palsy (depending on the position of
the cords) or cricoarytenoid joint disease.

Immuno compromised states lead to fungal infections. These include HIV,
Diabetes etc.

Patients taking long term steriods (inhalar or oral), antibiotics n those having
vitb12 and folic acid deficiecy are at a risk of having oral thrush.

Reference
Dr Khalid/Rabia


Q: 105 A 73yo male presents with enlarged cervical nodes. He has had recurrent
infections over the last year. His conjunctiva is pale. Choose the single cell type
you will find on the blood film.
a. Granulocyte without blast cells
b. Myelofibroblasts
c. Plasma cells
d. Mature lymphocytes



Clincher(s)
A
B

80
ID MERGED UNITY
February 17, 2016

C
D (Rabia) The diagnosis is CLL. It is CLL because of his age (73 yrs). Other
supportive features are cervical lymphadenpathy, recurrent infections
(mature but functionally defective lymphocytes), and pale conjunctiva
(anaemia).

It is a typical presentation of CLL with reurrent infections, symmetrical
lymphadenopathy, anemia. There can also be hepatosplenomegaly and
thrombocytopenia leading to patechae

On blood film there will be B cell lymphocytosis often with smudge cells.
There are mature but functionally impaired lymphocytes as they escape
apoptosis.

In bone marrow there is lymphocytic replacement of bone marrow cells.
Mainstay of treatment is chemotherapy.
E
KEY The key is D. Mature lymphocytes.
Additional
Information
Reference
Dr Khalid/Rabia


Q: 340 A pt complains of SOB, wheeze, cough and nocturnal waking. He has dry scaly
shin with rashes that are itchy. What is the single most likely dx?
a. Scabies
b. Eczema
c. Rheumatism
d. Dermatitis
e. Psoriasis

Clincher(s)
A
B
C
D
E
KEY B
Additional
Information
Reference
Dr Khalid/Rabia

81
ID MERGED UNITY
February 17, 2016

Q: 408 A pt from Africa comes with nodular patch on the shin which is reddish brown.
What is the
most probable dx?
a. Lupus vulgaris
b. Erythema nodosum
c. Pyoderma gangrenosum
d. Erythema marginatum
e. Solar keratosis




Clincher(s)
A
B [Causes of erythema nodosum: MOST COMMON CAUSES- i) streptococcal
infection ii) sarcoidosis. Other causes- tuberculosis, mycoplasma pneumonia,
infectious mononucleosis, drugs- sulfa related drug, OCP, oestrogen; Behcet’s
disease, CD, UC; lymphoma, leukemia and some others].

#Nodes are mostly on anterior aspect of shin
C
D
E
KEY The key is B. Erythema nodosum.
Additional
Information
Reference
Dr Khalid/Rabia


Q: 562 A 34yo female presents with a foul smelling discharge. What set of organisms
are we looking for to be treated here?
a. Chlamydia, gonorrhea
b. Chlamydia, gardenella
c. Chlamydia, gonorrhea, gardenella
d. Gonorrhea, gardenella
e. Gardenella only



Clincher(s)
A
B
C
D

82
ID MERGED UNITY
February 17, 2016

E Chlamydia is usually asymptomatic (no odour) and generally goes with
gonorrhea.


KEY e. Gardenella only
Additional BV will give the grey fishsmelling discharge
Information
Bacterial vaginosis and Trichomonas vaginalis give foul smelling discharge. In
BV its grey white fishy and in TV it can be greenish frothy fihy alongwith
vulvovginitis i-e strawberry cervix. The discharge of Chlamydia and Gonorrhea
is not foul smelling but gives dysuria.

Bacterial vaginosis (BV) describes an overgrowth of predominately anaerobic
organisms such as Gardnerella vaginalis.

Amsel's criteria for diagnosis of BV - 3 of the following 4 points should be
present
· thin, white homogenous discharge
· clue cells on microscopy: stippled vaginal epithelial cells
· vaginal pH > 4.5
· positive whiff test (addition of potassium hydroxide results in fishy odour)
Management
· oral metronidazole for 5-7 days
Reference
Dr Khalid/Rabia


Q: 567 A 78yo nursing home resident is revived due to the development of an
intensely itchy rash.
Exam: white linear lesions are seen on the wrists and elbows and red papules
are present on the penis. What is the most appropriate management?
a. Topical permethrin
b. Referral to GUM clinic
c. Topical betnovate
d. Topical ketoconazole
e. Topical selenium sulfide hyoscine




Clincher(s)
A
B
C
D
E

83
ID MERGED UNITY
February 17, 2016

KEY a. Topical permethrin
Additional Red papule on penis typical with wrist and elbow lesion goes with Scabies,
Information topical permethrin once wk and repeat if symptoms remain.

white linear lesions

Features
· widespread pruritus
· linear burrows on the side of fingers, interdigital webs and flexor aspects of
the wrist
· Nodules may develop. These occur particularly at the elbows, anterior
axillary folds,
penis, and scrotum.
· in infants the face and scalp may also be affected
· secondary features are seen due to scratching: excoriation, infection

Management
· permethrin 5% is first-line
· malathion 0.5% is second-line
· give appropriate guidance on use
· pruritus persists for up to 4-6 weeks post eradication
Reference
Dr Khalid/Rabia

Q: 569 A pt presents with gradual onset of headache, neck stiffness, photophobia and
fluctuating LOC. CSF shows lymphocytosis but no organism on gram stain. CT
brain is normal. What is the single most likely dx?
a. Hairy leukoplakia
b. TB
c. CMV infection
d. Candida infection
e. Cryptococcal infection




Clincher(s)
A
B
C
D
E
KEY b. TB
Additional TB as there is lymphocytosis and no organism on gram staining Zn staining or
Information AFB can detect mycobacterium TB

84
ID MERGED UNITY
February 17, 2016


Viral meningitis may be clinically indistinguishable from bacterial meningitis
but features may be more mild and complications (eg, focal neurological
deficits) less frequent. Any person presenting with suspected meningitis
should therefore be managed as having bacterial meningitis until proved
otherwise.

classic triad of fever, neck stiffness and a change in mental status was present
in only 44% of adults presenting with community-acquired acute bacterial
meningitis. However, 95% had at least two of the four symptoms of headache,
fever, neck stiffness and altered mental status.

Most patients with viral meningitis present with subacute neurological
symptoms developing over 1-7 days. Chronic symptoms lasting longer than
one week suggest meningitis caused by some viruses as well as TB, syphilis or
fungi.
Reference
Dr Khalid/Rabia

Bacterial Viral Tuberculous



Appearance Cloudy Clear/cloudy Slight cloudy, fibrin
web

Glucose Low (< 1/2 plasma) 60-80% of plasma Low (< 1/2 plasma)
glucose*

Protein High (> 1 g/l) Normal/raised High (> 1 g/l)

White cells 10 - 5,000 15 - 1,000 10 - 1,000


polymorphs/mm� lymphocytes/mm� lymphocytes/mm�



Q: 833 A 44yo man went on holiday to Sudan 5wks ago. He now presents with red
urine and fever. Exam: hepatomegaly. What is the most likely dx?
a. Malaria
b. Brucellosis
c. Leptospirosis
d. Schistosomiasis


Clincher(s) Holiday to African region and now complaints of red urine and fever with
hepatomagaly.
A
B Hx of farming and vet

85
ID MERGED UNITY
February 17, 2016

C Leptospirosis (also known as field fever,] rat catcher's yellows,] and pretibial
fever among others names) is an infection caused by corkscrew-shaped
bacteria called Leptospira. Signs and symptoms can range from none to mild
such as headaches, muscle pains, and fevers; to severe with bleeding from the
lungs or meningitis. If the infection causes the person to turn yellow, have
kidney failure and bleeding, it is then known as Weil's disease.[5] If it causes
lots of bleeding from the lungs it is known as severe pulmonary haemorrhage
syndrome.
D
E
KEY D
Additional
Information
Reference
Dr Khalid/Rabia Holiday in Sudan, hematuria, fever and hepatomegaly goes most
with Schissomiasis amongst the given option].


Q: 834 A 32yo homosexual comes with hx of weight loss. Fundoscopy reveals retinal
hemorrhages. What is the single most appropriate option?
a. Mycobacterium avium
b. CMV
c. Hemophilus influenze
d. NHL
e. Pneumocystic jerovic

Clincher(s) Young homosexual with hx of at loss. Fundoscopy reveals retinal
haemorrhages.
A Mycobacterium avium (M. avium).
Mycobacterium intracellulare (M. intracellulare
They are both opportunistic pathogens that affect the immunocompromised,
particularly HIV-positive individuals. They can also affect immunocompetent
people, especially those with pre-existing lung disease.
MAC can cause respiratory, gastrointestinal or disseminated infection in
patients with AIDS. It usually only affects the lungs in the immunocompetent
B
C No eye complication. Mainly pneumonia and meningitis
D No hx of lymph nodes mass and involvement of eye is not in NHL
E Causes atypical pneumonia
KEY B
Additional
Information
Reference Fundus pizza appearance in CMV retinitis.
Dr Khalid/Rabia

86
ID MERGED UNITY
February 17, 2016

Q: 1180 A 7yo child is being inv for TB. His parents don’t agree for taking a BAL. what
other sample will
show growth of the organism?


. a. Blood test 


. b. Throat swab


. c. Gastric washing

. d. Mantoux test


. e. CSF 



Clincher(s) Investigated for TB, refusal for BAL.
A No role of blood test.
B
C
D For screening we first do mantoux test.
E
KEY c- gastric washing

87
ID MERGED UNITY
February 17, 2016

Additional
Information


Reference
Dr Khalid/Rabia


Q: 1183 A 38yo man presents with acute infection of skin in the leg. Dx of cellutitis has
been made. What
meds should be prescribed?


a. Penicillin + Flucloxacillin 


b. Metronidazole + erythromycin

c. Vancomycin + metronidazole

d. Ceftriaxone + terbinafine 


e. Ceftriaxone + flucloxacillin



Clincher(s) Cellulitis and antibiotics
A
B

88
ID MERGED UNITY
February 17, 2016

C
D
E
KEY A- penicillin + flucloxacillin
Additional
Information


Reference
Dr Khalid/Rabia


Q: 1427 A 37yo male pt who recently returned back to UK from UAE attends the OPD
with complaint of dry cough, breathlessness and anorexia. According to him he
had flu like symptoms a week ago. He is slightly confused. Inv: lymphopenia &
decreased Na+. CXR: bi-basal consolidation. What is the single most likely
causative organism?


a. Legionella


89
ID MERGED UNITY
February 17, 2016

b. Chlamydia pneumoniae

c. PCP


d. Viral pneumonia


e. Chlamydia psittaci




Clincher(s) Travel history to dry and humid area, bibasal consolidation with flu like
symtpoms, hyponatremia
A
B
C
D
E
KEY A legionella
Additional
Information


Reference
Dr Khalid/Rabia


Q: 1428 A 20yo student came to the OPD with complains of headache, malaise, dry
cough, joint pain and vomiting. Exam: temp=39C. CXR: patchy consolidation.
What is the single most likely causative organism?


a. Pneumococcal pneumonia

b. Mycoplasma

c. Klebsiella


d. Streptococcus

e. PCP



Clincher(s)
A Pneumococcal pneumonia common in elderly, alcholics, post-splenectomy

90
ID MERGED UNITY
February 17, 2016

patients. CXR shows lobar consolidation.
B
C Klebseilla cause cavitationg pneumonia in elders and diabetics.
D
E PCP in immunocompromised with normal or bilateral perihilar interstitial
shadowing on CXR.
KEY B- Mycoplasma pneumonia.
Additional
Information


Reference
Dr Khalid/Rabia


Q: 1449 Which virus is transmitted by the fecal-oral route?

a. Hep C


b. Coxsackie virus


c. Dengue

d. None of the above



Clincher(s)
A Blood borne virus.
B
C Spread by mosquito bite
D
E
KEY C- Coxsackie virus.
Additional
Information
Reference
Dr Khalid/Rabia

Q: 1451 A 7yo child presents with lesions on the trunk. Exam: some lesions are weeping
and others are crusted with a red base. What is the causative organism?


91
ID MERGED UNITY
February 17, 2016

a. Herpes simplex


b. Varicella zoster

c. Rubella virus


d. Herpes zoster




Clincher(s) Lesions on the trunk, blisters in different stages with weeping lesions.
A
B
C
D
E
KEY B- Varicella zoster(chicken pox)
Additional
Information


Reference
Dr Khalid/Rabia

Q: 1622 A pt. comes back from India and presents with night sweats and
lymphadenopathy. XR:
Cavitation. What investigation should be done next?


92
ID MERGED UNITY
February 17, 2016

a. CT scan

b. AFB stain


c. Blood culture


d. Bronchoscopy




Clincher(s) Night sweats and lymphadenopathy with cavitation, recent travel history to
the India.
A
B
C
D
E
KEY B- AFB stain for tuberculosis.
Additional
Information
Reference
Dr Khalid/Rabia


Q: 1627 A healthy baby boy is born at term to a woman who was unwell with
confirmed acute hep B
during pregnancy. The mother is very concerned that
she may have infected the baby with hep B. What SINGLE preventative
intervention should be given to the baby?

a. Full course of hepatitis B vaccine


b. Hepatitis B immunoglobulin alone


c. Hepatitis B vaccine and hepatitis B immunoglobulin

d. Hepatitis B vaccine as single dose




Clincher(s) Mother hepatitis B and treatment plan for newborn
A
B
C
D
E
KEY C. Hepatitis B vaccine and hepatitis B immunoglobulin immediately.
Additional

93
ID MERGED UNITY
February 17, 2016

Information
Reference
Dr Khalid/Rabia


Q: 1642 A 19yo woman has had progressive bilateral iliac fossa pain and dyspareunia
for 3days. She has
an offensive vaginal discharge and feels unwell and
feverish. Her temp=39C. An initial antimicrobial regimen is commenced. What
SINGLE set of organisms are the most appropriate for the antimicrobial
regimen to cover?


a. Neisseria gonorrhoeae and Candida albicans


b. Neisseria gonorrhoeae and Candida albicans and Gardnerellavaginalia


c. Neisseria gonorrhoeae and Chlamydia trachomatis


d. Neisseria gonorrhoeae and Chlamydia trachomatis and Candida albicans


e. Neisseria gonorrhoeae and Chlamydia trachomatis and Gardnerellavaginalis



Clincher(s) H/o dyspareunia and bilateral lower abdominal pain with vaginal discharge.
A
B
C
D
E
KEY C. Neisseria gonorrhea and chlamydia trachomatis. Patient is suffering from
PID(pelvic inflammatory disease).
Additional
Information
Reference
Dr Khalid/Rabia


Q: 1660 A 17-year-old boy is diagnosed with scabies. Which of the following statements
regarding
scabies is correct?


a. Is best treated by salicylate emulsion 


b. It can be spread by a droplet infection


c. It causes itchiness in the skin even where there is no obvious lesion to be


seen


94
ID MERGED UNITY
February 17, 2016

d. It is caused by Staphylococcus aureus
e. Typically affects the face





Clincher(s) Diagnosed case of scabies.
A
B
C
D
E
KEY C. it causes itchness in the skin even where there is no obivious lesions ti be
seen.
Additional
Information
Reference
Dr Khalid/Rabia

Q: 1662 An association with HPV is a most characteristic feature of?
A. Torus.
B. Exotosis.
C. Pleomorphic adenoma.
D. Verruca vulgaris.
E. Fibroma.
F. Epulis fissuratum.
G. Mucocele.
H. Pyogenic granuloma.
I. Parulis.
J. Ranula.


clinchers Human papilloma virus
A Congenital growth in soft palate
B Growth of bone.
C
D
E
Key D- Verruca vulgaris(plantar wart)
Additional F) growth around the dental implant. Gum hyper trophy.
Information
Reference
Dr.Khalid/Rabia

Q: 1674 An 18year male works in a company where lunches are often catered. One
day, the water at the company facility is not working, but they manage to have
the lunch anyway. 2 weeks later, he becomes sick. He develops anorexia,

95
ID MERGED UNITY
February 17, 2016

nausea, malaise and jaundice. During the course of next 4 weeks, 7 people
who shared in the lunch become ill with similar symptoms. After a few weeks,
each of the 7 people completely recovers and they replace their caterer. What
is a likely diagnosis?
A. Pancreaic carcinoma.
B. Hemochromatosis.
C. Laennec’s cirrhosis.
D. Hep A.

clinchers Catering labourer, jaundice, anorexia, nausea, malaise.
A
B
C . Laennec's cirrhosis can be associated with inflammatory polyarthritis, most
commonly affecting the shoulders, elbows and knees. Osteoporosis, soft tissue
swelling in peripheral joints and sometimes calcific periathritis are seen.
D
E
Key D- Hepatitis A.
Additional
Information
Reference
Dr.Khalid/Rabia

Q: 1692 A 22yo has had recent chickenpox. He now presents with confusion. He is noted to have
low
urine output and large petechiae all over his body. CXR: a large patch of consolidation is
seen.
The management of choice should be :


a. Ventilatory support


b. Open surgical debridement


c. Resection of superficial
petechiae with wide margin


d. Booster vaccine


e. TENS


f. Lontophoresis


g. Nephrostomy


h. Oral Corticosteroids

i. Brivudin


j. IV acyclovir


96
ID MERGED UNITY
February 17, 2016

clinchers Diagnosed case of chicken pox with classical signs.
A
B
C
D
E
Key J- IV acyclovir.
Additional
Information


Reference
Dr.Khalid/Rabia

Q: 1694 Syphilis typically causes


a. Lymphogranuloma Venereum


b. Testicular Torsion


c. Hydrocele


d. Epididymitis


97
ID MERGED UNITY
February 17, 2016

e. Seminoma


f. Mature teratoma


g. Varicocele


h. Lymphoma


i. Orchitis


j. Spermatocele




clinchers Complications of syphilis
A
B
C
D
E
Key i. Orchitis.
Additional
Information
Reference
Dr.Khalid/Rabia






98

S-ar putea să vă placă și